76
Al completar la unidad, alumnas y alumnos: reconocen situaciones de la vida diaria que se explican en base a la presión, como por ejemplo, el poder tomar bebida con una pajilla; replican el modelo atómico y molecular para explicar el comportamiento de los fluidos y los efectos de la presión; comprenden el funcionamiento de diversos sistemas hidráulicos, como por ejemplo, el de los frenos de los automóviles; en base al principio de Arquímedes, comprenden las condiciones de flotabilidad, por ejemplo, de los barcos; explican el fenómeno de la capilaridad y reconocen su importancia, por ejemplo, a nivel biológico: utilizan el principio de Bernoulli para explicar, por ejemplo, la sustentación de los aviones; reconocen en las leyes que describen el movimiento de un cuerpo en un fluido una explicación para la velocidad límite que alcanza, por ejemplo, una gota de lluvia en la atmósfera; describen las principales características físicas del sistema cardiovascular; conocen aspectos biográficos de quienes desarrollaron la física de los fluidos.

FLUIDOS - FISICA

  • Upload
    iopdsj

  • View
    399

  • Download
    8

Embed Size (px)

Citation preview

Page 1: FLUIDOS - FISICA

Al completar la unidad, alumnas y alumnos:

• reconocen situaciones de la vida diaria que se explican en base a la presión, como por ejemplo, el poder tomar bebida con una pajilla;

• replican el modelo atómico y molecular para explicar el comportamiento de los fluidos y los efectos de la presión;

• comprenden el funcionamiento de diversos sistemas hidráulicos, como por ejemplo, el de los frenos de los automóviles;

• en base al principio de Arquímedes, comprenden las condiciones de flotabilidad, por ejemplo, de los barcos;

• explican el fenómeno de la capilaridad y reconocen su importancia, por ejemplo, a nivel biológico:

• utilizan el principio de Bernoulli para explicar, por ejemplo, la sustentación de los aviones;

• reconocen en las leyes que describen el movimiento de un cuerpo en un fluido una explicación para la velocidad límite que alcanza, por ejemplo, una gota de lluvia en la atmósfera;

• describen las principales características físicas del sistema cardiovascular;

• conocen aspectos biográficos de quienes desarrollaron la física de los fluidos.

Page 2: FLUIDOS - FISICA

113

Un hipopótamo requiere alimentarse de la vegetación que crece en el fondo de un estanque. Cuando el hipopótamo vadea en el estanque, flota. ¿Por qué flota en lugar de hundirse hasta el fondo del estanque?

Page 3: FLUIDOS - FISICA

Física 3° Año Medio114

Fluidos

Antes de empezar...

1 ¿En cuántos estados o fases se encuentra la materia conocida?

2 ¿Qué diferencia a un sólido de un fluido?

3 ¿Si un gas y un líquido son fluidos, que diferencia entonces a un gas de un líquido?

4 ¿Cómo se llama la relación por cuociente entre la masa y el vo-lumen de un cuerpo?

5 ¿Cómo se llama la relación entre la masa de una hoja de cuaderno y su área?

6 ¿Cuál es la magnitud que corres-ponde a la medida en que una fuerza se distribuye sobre el área de una superficie?

7 ¿Se pueden comprimir los flui-dos?

8 ¿Qué ocurre con el nivel del agua cuando te sumerges en una piscina?

9 ¿Qué ocurre con el peso de tu cuerpo cuando nadas en una piscina?

10 ¿Puede una aguja de acero “flotar” en la superficie del agua?

11 ¿Qué tienen en común las burbujas de jabón con las gotas de rocío en una tela de araña?

12 ¿Cómo llega el agua desde las raíces de un árbol hasta sus hojas más altas?

13 ¿Cuándo un líquido moja una superficie? ¿Todos los líquidos mojan cualquier superficie?

“Proposición 6: si un sólido es más ligero que un fluido y se sumerge fuertemente en él, el sólido será llevado hacia arriba por una fuerza igual a la diferencia entre su peso y el peso del fluido desplazado”.

Arquímedes de Siracusa (287 a. C. – 212 a. C.), físico y filósofo griego.

Capítulo 3

Page 4: FLUIDOS - FISICA

115

Capítulo 3: Hidrostática

Indagación N°13¿Cómo puede un faquir acostarse en una cama de clavos?

PARTE I: Trabajo personal

En física es habitual hacer uso de modelos simplificados de los fenómenos que ocurren en la naturaleza. En la aplicación de las leyes de Newton, por ejemplo, los cuerpos en general son tratados como si fueran partículas u objetos puntuales. Así, podemos hablar de objetos puntuales y, a veces, de objetos extensos.

a) ¿En qué circunstancias consideras que un globo inflado puede ser modelado como una partícula y en qué circunstancias, como un objeto extenso?

b) ¿Cuál es el procedimiento más efectivo que imaginas para reventar un globo?

c) Si aprietas un globo contra una cama de clavos, como la de un faquir (imagen 5.1), ¿se revienta?

PARTE II: Trabajo en equipo

Junto a un compañero o una compañera, contrasten sus respuestas y argumenten a favor o en contra de ellas.

A continuación, elaboren una hipótesis en conjunto que dé respuesta a la tercera pregunta.

a) Registren la hipótesis en sus cuadernos e identifiquen cuáles son las variables observables que pueden medir y/o controlar.

b) Una vez planteada su hipotesis, diseñen un procedimiento experimental que les permita ponerla a prueba, para evaluar si es una explicación aceptable o debe ser descartada. Dibujen su montaje experimental y describan brevemente, pero con precisión, el procedimiento que sugieren.

Procuren que el procedimiento experimental propuesto sea factible de realizar en una hora de clases; es decir, que incluya el uso de materiales de fácil adquisición o construcción y tiempos razonables para la obser-vación y el análisis de sus resultados.

c) Para finalizar, elaboren un informe de dos páginas según las indica-ciones que les dé su profesor(a).

Imagen 5.1

Recuerden que una hipótesis es una explicación posible que se supone cierta hasta que pueda ser contrastada empíricamente. Por esta razón, es fundamental que la hipótesis se refiera a un número reducido de variables observables y de algún modo medibles, que eventualmente pueden ser controladas en un experimento.

Page 5: FLUIDOS - FISICA

Física 3° Año Medio116

Fluidos

Indagación N°14

¿Cómo clavar un clavo?

PARTE I: Trabajo personal

Cuando el carpintero clava un clavo en un trozo de madera, pone la punta del clavo hacia la madera y le pega luego con el martillo, en lo que se llama cabeza del clavo. Esto le permite ejecutar un buen clavado. Sin embargo, si se equivoca y pone el clavo al revés, por más fuerte que lo golpeé con el martillo, el clavo no entra.

De acuerdo a lo anterior, ¿por qué el clavo no entra en la madera al ponerlo de cabeza en la tabla, si la fuerza aplicada por el carpintero es la misma o incluso mayor?

a) Plantea una hipótesis que dé respuesta a esta pregunta y regístrala en tu cuaderno.

PARTE II. Diálogo con argumentos

Reúnete con un compañero o compañera para compartir sus hipótesis. Comenten y argumenten a favor o en contra de ella. Luego, sigan con atención los dos casos que mostrará su profesor(a) y respondan en su cuaderno las siguientes preguntas:

a) ¿Cuál es la diferencia entre los dos casos mostrados?

b) ¿Cómo se compara la fuerza aplicada por los chinches o tachuelas en el primer caso, respecto al segundo?

c) ¿Cómo se compara el área de contacto de los chinches o tachuelas con el globo en el primer caso, respecto al segundo?

d) ¿Cuál de los dos casos es más efectivo para reventar el globo? ¿Por qué?

PARTE III. Trabajo en equipo

En esta parte de la actividad, junto a tu compañero(a) realizarán un sencillo experimento. Necesitan los siguientes materiales: un trozo de madera trupán de 10 cm x 10 cm aproximadamente, un martillo, un alfiler, un chinche, 4 clavos de distintas medidas (media pulgada: (½)"; una pulgada: (1)"; tres pulgadas: (3)"; 4 pulgadas: (4)") y un perno de 3 pulgadas: (3)".

Primero, uno(a) de ustedes, usando el martillo, clava en la madera el alfiler, el chinche y los 4 clavos. Observen la fuerza aplicada en cada caso. A continuación, intentan aplicar la misma fuerza que en el caso de un clavo, para “clavar” el perno en la madera, también usando el martillo. Intenten introducir el perno en la madera, aplicando golpes cada vez más fuertes con el martillo sobre la cabeza del perno, hasta lograr insertarlo un poco.

a) ¿Cómo es, comparativamente, la dificultad para clavar cada objeto en la madera? Comparen sus observa-ciones para el caso de los clavos, el alfiler y el perno.

b) ¿Qué papel cumple el área de la superficie de contacto, entre los objetos y la madera?

c) Comparen sus respuestas con sus hipótesis iniciales.

Page 6: FLUIDOS - FISICA

117

Capítulo 3: Hidrostática

Presión y principio de Pascal

La materia se presenta en distintos estados o fases, cuyas propie-dades y características son diferentes. Históricamente, se recono-cieron tres estados, de acuerdo a distinciones cualitativas entre sus propiedades macroscópicas. Actualmente, las distinciones entre estados de la materia están basadas en diferencias en sus interacciones moleculares y así se pueden reconocer por lo menos cuatro estados diferentes:

• Sólido. Es el estado en el cual la materia tiene forma y volumen definidos. En este caso, la atracción intermolecular mantiene a las moléculas en posiciones relativas fijas.

• Líquido. Es el estado en el que la materia mantiene un volumen definido, pero cambia su forma de acuerdo a su contenedor. En este caso, la atracción entre las moléculas logra mantenerlas relativamente próximas, pero no lo suficiente para fijar sus posiciones relativas.

• Gas. Es el estado en el que la materia se expande hasta ocupar cualquier volumen disponible. En este caso, las moléculas están relativamente separadas y la atracción intermolecular tiene un efecto despreciable en su movimiento.

• Plasma. Se trata de una sustancia compuesta por una colección de partículas libres con carga eléctrica.

A las sustancias en estado gaseoso o en estado líquido les llama-mos fluidos. Esta denominación se debe a que, en determinadas circunstancias, este tipo de sustancia tiene la propiedad de escurrir o fluir, ya que su forma se adapta cualquier contenedor sólido.

En esta sección nos concentraremos en el estudio de las propiedades mecánicas de los fluidos que se encuentan en reposo.

¿Cuál es el estado de la materia más abundante en el Universo?

Figura 5.1. Los tres estados clásicos de la materia. A nivel molecular se dife-rencian por su grado de cohesión.

Los estados de la materia también se pueden definir en términos de transiciones de fase, las que indican un cambio de estructura interna y pueden ser reconocidas por abruptos cambios en las propiedades. De este modo, el número de estados diferentes crece significativamente.

Figura 5.2. Un fluido gaseoso no solo se adapta al recipiente que lo contiene, sino que también se puede comprimir.

Figura 5.3. Un fluido líquido puede cambiar su forma, pero no puede ser comprimido (a temperatura cons-tante).

Sólido Líquido Gas

5

Sección 5: Presión y principio de Pascal

Sección

Page 7: FLUIDOS - FISICA

Física 3° Año Medio118

Fluidos

Líquidos y gases en el UniversoAunque los líquidos y gases son fluidos, la distinción entre fluidos y solidos no es completamente obvia.

Para hacer una distinción rigurosa, es necesario evaluar una propiedad de las sustancias conocida como viscosidad, que estudiaremos en la siguiente sección. Un caso bien documentado, por ejemplo, es el de una sustancia muy común en nuestras ciudades y carreteras, conocida como asfalto.

El asfalto se puede encontrar de manera natural en depósitos de petróleo crudo, pero se obtiene también fácilmente como un subproducto en las refinerías petroleras. Se trata de una sustancia que al tacto parece dura, pero que en realidad puede fluir. Esto lo demuestra el experimento de la gota de asfalto, que se empezó en 1927 y ¡todavía continua! Consiste en dejar caer gotas de asfalto desde un embudo a otro recipiente. 70 años después de iniciar el experimento, cayó la octava gota de asfalto, y actualmente sigue formándose la gota número 9.

Pero no solo los líquidos que parecen sólidos son interesantes en el mundo de los fluidos. De hecho, gran parte del Universo está hecha de fluidos. La atmósfera y los océanos de la Tierra, son gases y líquidos, respectivamente. Incluso la roca y el metal a elevadas temperaturas son fluidos en las profundidades de la Tierra. En el Sistema Solar, Júpiter, Saturno, Urano y Neptuno son planetas gigantes gaseosos, constituidos principalmente de gases o gases comprimidos en estado líquido. Sin embargo, la materia visible más abundante en el Universo se encuentra en forma de plasma en las estrellas y en las nubes interestelares.

La forma que adoptan los fluidos está determinada por las fuerzas externas que actúan sobre ellos. En el caso de la atmósfera terres-tre, por ejemplo, el gas adapta la forma de una cáscara esférica que rodea al planeta, debido a la acción de la fuerza de gravedad. Lo mismo ocurre en el caso de las estrellas y planetas gaseosos, ya que su simetría esférica obedece a la acción de la fuerza de gravedad.

En el espacio interestelar, los fluidos tienden a adoptar una forma esférica, como las gotitas microscópicas de agua que se forman en las nubes. Sin embargo, y a nivel terrestre, un líquido en un vaso, por ejemplo, adquiere su forma por la acción de la fuerza de gravedad y de las paredes del vaso.

En ocasiones, se ha considerado que el vidrio es un ejemplo de una sustancia comunmente considerada sólida, pero que en realidad fluye en forma muy lenta como si fuera un líquido. Aparentemente, esta idea surge porque se ha observado que los vidrios de antiguas catedrales tienen un espesor claramente mayor en su parte inferior que en la superior. Sin embargo, esta información no está estadísticamente validada, y tampoco existe consenso científico en considerar al vidrio como un líquido. Sí, en cambio, hay consenso en considerar la estructu-ra molecular del vidrio como la de un sólido amorfo.

Figura 5.4. Un planeta fluido. El planeta Júpiter tiene la cuarta parte de la densidad de nuestro planeta, lo que permite inferir que es mucho más gaseoso en su estructura interna que la Tierra.

Los objetos físicos son sensibles al cambio de magnitud de algunas de sus propiedades. Por ejemplo, un sólido, como un riel de la línea férrea o una puerta, al experimentar cambios de temperatura, experimenta dilatación térmica. Es decir, aun siendo un sólido pierde su propiedad de tener volumen fijo. En esta sección no consideramos los efectos de los cambios de temperatura en los fluidos, ni en los sólidos que actúan como contenedores.

Page 8: FLUIDOS - FISICA

119

Capítulo 3: Hidrostática

Conceptos preliminares

EXTENSIÓN Y DIMENSIONALIDAD DE LOS OBJETOSEn general, los objetos que nos rodean, independientemente de su forma, son cuerpos que ocupan un volumen determinado, dis-tribuido en las tres dimensiones espaciales.

Sin embargo, algunos objetos pueden presentar una distribución geométrica que privilegia una, dos o tres dimensiones espaciales. Por ejemplo, una varilla cilíndrica, cuya longitud es mucho mayor que su diámetro, puede ser modelada como un objeto unidimensional. La tapa de un cuaderno, en cambio, cuyas dimensiones significativas son el largo y el ancho, en comparación con el espesor del cartón, puede ser modelada como un objeto bidimensional.

En esta sección, solo consideramos objetos cuya masa se distribu-ye en una estructura geométrica que tiene dos o tres dimensiones principales, es decir: superficies y volúmenes.

Figura 5.6. Un objeto que tiene su masa distribuida principalmente en una sola dimensión, como las agujas o un trozo de hilo, podemos considerarlo unidimensional.

Figura 5.7. Un objeto que tiene su masa distribuida principalmente en dos dimensiones, como la lámina de vidrio, podemos considerarlo bidi-mensional.paralelepípedo

cono esfera

V a b c= ⋅ ⋅

V r h= ⋅ ⋅π 2

3V r= ⋅4

33π

Figura 5.5. Parámetros para el cálculo del volumen (V) de algunos cuerpos geométricos regulares.

¿Por qué, a pesar de su espesor, el vidrio que se muestra en la Figura 5.7 puede modelarse como una superficie?

cilindro

V r h= ⋅ ⋅π 2

h

r

c

ab

cubo

V a= 3

a

aa

h

r

r

Sección 5: Presión y principio de Pascal

Page 9: FLUIDOS - FISICA

Física 3° Año Medio120

Fluidos

Figura 5.8. Un globo aerostático se eleva porque la densidad del fluido en su interior es menor que la densidad del fluido en su exterior. ¿Cuál es el fluido dentro y fuera del globo aerostático? ¿Por qué tienen diferente densidad?

DENSIDAD VOLUMÉTRICAAl relacionar la masa de un objeto con sus dimensiones geomé-tricas, se obtiene una magnitud conocida como densidad (ρ). La densidad de un cuerpo se puede determinar de distintas formas, dependiendo de la geometría del objeto y su dimensionalidad.

Así, al considerar un objeto cuya masa se encuentra distribuida en una sola dimensión principal, como el caso de la varilla larga, hablamos de densidad lineal. Cuando se trata de un cuerpo cuya masa está distribuida principalmente en dos dimensiones, hacemos uso de una densidad superficial. Cuando la masa se distribuye sin privilegiar ninguna dimensión, como el caso de una roca o un cilindro, consideramos la densidad volumétrica.

Para el estudio de los fluidos, centramos nuestra atención en la comprensión de la idea de densidad volumétrica. Por eso, en adelante, cada vez que nos refiramos a la densidad de un cuerpo, estaremos hablando de su densidad volumétrica a menos que se indique otra cosa.

De acuerdo a esto, la densidad de un cuerpo cualquiera es una magnitud escalar, cuya unidad de medida en el Sistema Interna-cional es Kg

m3, y se determina de la siguiente forma:

ρ =mV

(5.1)

En la ecuación 5.1 m es la masa y V es el volumen del cuerpo.

Ejemplo 1

Un ladrillo de 5 kg tiene las siguientes dimensiones: 30 cm de largo, 10 cm de ancho y 5 cm de alto.

a) ¿Cuál es el volumen del ladrillo y su densidad?

a: De acuerdo al enunciado, el ladrillo puede ser modelado como un paralelepípedo. De esta manera, el volumen del ladrillo es:

V a b cV m m mV m

= ⋅ ⋅= ⋅ ⋅=

0 3 0 1 0 050 0015 3

, , ,,

ρ es la letra griega “rho”.

Page 10: FLUIDOS - FISICA

121

Capítulo 3: Hidrostática

Para calcular la densidad volumétrica, usamos la ecuación (5.1), reemplazando el volumen y la masa conocida del ladrillo:

ρ

ρ

ρ

=

=

=

mV

Kgmkgm

50 0015

3 333 3

3

3

,

,

Es decir, de acuerdo a la Tabla 5.1, podemos decir que el ladrillo tiene una densidad 3 veces mayor que la del agua.

Como has aprendido en cursos anteriores, agua, aceite y mercurio son líquidos inmiscibles, es decir, no pueden mezclarse entre sí. Si se pone en un vaso una parte de cada una de estas sustancias, ¿cuál sería su distribución, por capas, en el interior del vaso?

LA PRESIÓNEn el estudio del movimiento, en segundo año medio, y también en el capítulo anterior sobre el movimiento circular, has aprendido en qué consiste una fuerza y cuáles son sus efectos. Hemos visto que la fuerza siempre es una interacción entre dos objetos.

Por lo general, al modelar las interacciones, consideramos los ob-jetos como si fueran partículas puntuales, de modo que las fuerzas actúan en un punto específico de cada objeto. Este punto se llama centro de masa. Es decir, es en el centro de masa del objeto donde operacionalmente se aplica una fuerza.

Sin embargo, cuando dos objetos extensos interactúan mediante una fuerza, de manera que una gran cantidad de puntos de sus su-perficies están en contacto, decimos que los objetos ejercen presión entre sí. La presión es, entonces, una fuerza que se distribuye en una superficie y actúa en un área determinada.

De acuerdo a esto, la presión se define del siguiente modo:

P F

A= (5.2)

Donde F es el módulo de la fuerza perpendicular a la superficie cuya área de contacto es A.

Tabla 5.1. Densidad volumétrica de algunas sustancias sólidas y líquidas. Estos valores varían ligeramente con la temperatura, porque el volumen de una sustancia depende de la temperatura a la que se encuentre.

El centro de masa de un sistema es el punto donde se supone concentrada toda la masa del sistema. Este concepto se utiliza para análisis físicos en los que no es importante considerar la distribución de masa. Un sistema, en este sentido, puede ser un cuerpo extenso, o una colección de varios cuerpos masivos.

Sustancia Densidadkgm3

Densidadg

cm3

Agua (a 4°C) 1 000 1,0000Aceite 920 0,92Gasolina 680 0,6800Plomo 11 300 11,300Acero 7 800 7,8000Mercurio 13 600 13,6000Madera 90 0,9000Aire 1,3 0,0013

Figura 5.9. El globo no revienta debido a que la fuerza que se aplica se distri-buye en todos los puntos de contacto. Entre mayor es la superficie de contacto menor es la presión.

Sección 5: Presión y principio de Pascal

Page 11: FLUIDOS - FISICA

Física 3° Año Medio122

Fluidos

Figura 5.11. La presión que ejerce la suela del calzado sobre el suelo es mu-cho mayor en el taco que en la planta, ya que mientras menor es el área de contacto mayor es la presión.

De acuerdo a lo anterior, la unidad de medida corresponde a Nm2

, que recibe el nombre de pascal [Pa]:

PaNm

=

2

(5.3)

X Y Z

Figura 5.10. Un ladrillo apoyado sobre cada uno de sus lados. ¿En qué caso el peso del ladrillo ejerce mayor presión sobre el suelo?

Ejemplo 2

Supongamos que la masa del ladrillo de la Figura 5.10 es de 5 kg, y tiene las siguientes dimensiones: 30 cm de largo, 10 cm de ancho y 5 cm de alto.

a) Entre los lados del ladrillo, ¿cuál es el área menor y el área mayor?

b) ¿Cuál es la presión que ejerce el ladrillo al estar apoyado en cada uno de esos lados?

a: Las áreas de los tres lados del ladrillo son:

A m m

A m

A m m

A

X

X

Y

Y

= ⋅

=

= ⋅

0 1 0 05

0 005

0 3 0 05

2

, ,

,

, ,

==

= ⋅

=

0 015

0 3 0 1

0 03

2

2

,

, ,

,

m

A m m

A mZ

Z

Por lo tanto, las caras de menor área son las superficies de apoyo del ladrillo en los casos X e Y de la Figura 5.10.

Page 12: FLUIDOS - FISICA

123

Capítulo 3: Hidrostática

b: Para encontrar la presión ejercida en cada caso, necesita-mos precisar la fuerza que se aplica sobre el suelo. Esta fuerza corresponde al peso. Aproximando la aceleración de gravedad como g

ms

= 102

, de acuerdo a la ecuación (5.2), la presión ejercida por el área menor es:

PFA

PmgA

Pkg m

sm

PNm

P

=

=

=⋅

=

=

5 10

0 005

10 000

10

2

2

2

,

0000Pa

Análogamente, la presión ejercida por el área mayor es:

Pkg m

sm

PNm

P Pa

=⋅

=

=

5 10

0 03

1 666 7

1 666 7

2

2

2

,

,

,

Imagina que un compañero te empuja con mucha fuerza con la palma de su mano extendida y, luego, te pincha con una aguja, pero con una pequeña fuerza muy leve. ¿Por qué te duele el pinchazo y no el empujón si te lo dio con más fuerza?

Es importante observar que la fuerza y la presión son magnitudes diferentes. Podemos obtener una presión muy grande a partir de una fuerza relativamente pequeña, haciendo que el área sobre la que se aplica la fuerza sea pequeña, como es el caso de una aguja o el taco de la Figura 5.11.

También podemos producir una presión pequeña a partir de una fuerza grande, aumentando el área sobre la que actúa la fuerza, como es el caso de los esquíes en la nieve o del globo en la Figura 5.9.

Sección 5: Presión y principio de Pascal

Page 13: FLUIDOS - FISICA

Física 3° Año Medio124

Fluidos

Actividad de profundización

¿De qué depende la flotación de un objeto?

Reúnete con 4 ó 5 compañeros y compañeras y formen un equipo de trabajo.

Para realizar esta actividad, necesitan lo siguiente: 100 cm3 de agua, 100 cm3 de aceite y 100 cm3 de alcohol de quemar (etanol). También necesitan una botella plástica de bebida de 0,5 litros. Por último, incluyan en la lista: una nuez (con cáscara), una mandarina, 10 chinches metálicos, 1 bolita de vidrio y/o de acero (como la de un rodamiento). Si es posible, consideren también un cubo de hielo y otro de aceite congelado.

a) Reflexionen sobre esta pregunta: ¿cómo se relaciona la flotación de un cuerpo con su densidad? Como equipo, planteen una hipótesis para responder.

A continuación, realicen el siguiente experimento. Primero, midan la masa y el volumen de cada líquido con precisión, haciendo uso de los instrumentos pro-porcionados por tu profesor(a). Luego, respondan:

b) ¿Cuál es la densidad de cada uno de los líquidos?

c) ¿Cómo pueden medir el volumen del 1 chinche metálico o de la nuez? ¿Cuál es la densidad de cada uno de los objetos?

A continuación, corten el cuello de la botella, de manera que la abertura superior tenga 4 cm de diámetro aproximadamente. Introduzcan lentamente en su interior cada uno de los líquidos en el siguiente orden: agua, aceite y etanol.

d) Describe en tu cuaderno qué observas una vez que todos los líquidos se han vertido en la botella.

e) ¿Qué papel desempeña la densidad de los líquidos en lo que observas?

A continuación, dejen caer dentro de la botella, uno a uno, los objetos que con-siguieron. Antes de depositarlos en la botella, predigan hasta donde se hundirá cada objeto. Luego, observen.

f) Discutan sus respuestas y compárenlas con la hipótesis que plantearon.

Para finalizar la actividad, preparen un informe sobre su trabajo según las indi-caciones de su profesor(a) y luego presenten a sus compañeros(as) cuáles fueron sus hallazgos.

Page 14: FLUIDOS - FISICA

125

Capítulo 3: Hidrostática

Evaluación intermediaPARTE I: Problema de planteamiento

1 Observa la siguiente imagen:

a) En la imagen se observan los pies de tres niños que caminan en zancos hechos de tarros con arena en el interior. El radio de un tarro de es de 0,07 m. Si la masa de un niño es de 50 kg y la de cada uno de sus zancos es de 1 kg, ¿cuál es la presión que se ejerce sobre el suelo cuando el niño está parado en un pie?

PARTE II: Análisis

2 A partir del problema anterior, ¿cuánto aumenta la presión de un zanco sobre el suelo si el radio del tarro se reduce a la mitad?

Indagación N°15¿Cómo cambia la presión en el interior de un líquido con la profundidad?

Para responder la pregunta planteada en el título de esta actividad, se propone la si-guiente hipótesis:

La presión en el interior de un líquido es siempre la misma a cualquier profundidad, es decir, la presión es constante.

¿Cómo podemos poner a prueba esta hipótesis?

a) Junto a un compañero o una compañera, diseñen un procedimiento experimental que les permita, a través de un modelo, poner a prueba la hipótesis para evaluar si es una explicación aceptable o debe ser descartada. Dibujen su montaje experimental y describan brevemente, pero con precisión, el procedimiento que sugieren.

Procuren que el procedimiento experimental propuesto sea factible de realizar en una hora de clases; es decir, que incluya el uso de materiales de fácil adquisición o cons-trucción y tiempos razonables para la observación y el análisis de sus resultados.

b) Para finalizar, elaboren un informe de dos páginas según las indicaciones que les dé su profesor(a).

Sección 5: Presión y principio de Pascal

Recuerda que un modelo es una representación simplificada del fenóme-no que se intenta explicar, que incorpora sus prin-cipales características y, en especial, las variables medibles.

Page 15: FLUIDOS - FISICA

Física 3° Año Medio126

Fluidos

Presión hidrostáticaAunque la presión no tiene una dirección específica, la fuerza que la produce sí la tiene. En el caso de un fluido que ejerce presión sobre una superficie, hay una fuerza neta que siempre está dirigida en una dirección perpendicular a la superficie. Analicemos esta idea considerando la Figura 5.12.

Cuando sumergimos un dedo en un vaso con agua o cuando nos sumergimos en una piscina, podemos percibir la fuerza de contacto del agua en cada punto sumergido de nuestro cuerpo. Al igual que en la Figura 5.12(a), las fuerzas del líquido actúan sobre nuestra piel apuntando en todas direcciones.

Fijemos nuestra atención en el punto medio de una de las caras del bloque triangular. En ese lugar, como en cualquier otro lugar de las superficies del bloque, actúan fuerzas que apuntan en todas direcciones. El esquema muestra que las componentes paralelas a la superficie del objeto se anulan entre sí (las componentes paralelas de las fuerzas azules y verdes), y sólo queda un fuerza neta, perpendicular a cada lado, que es la suma de las componen-tes perpendiculares de las fuerzas del líquido, actuando sobre las superficies del bloque.

Como muestra la Figura 5.12(b), la fuerza ejercida por el líquido sobre las superficies del objeto es siempre perpendicular a ellas. Lo mismo sucede en las paredes del recipiente.

Figura 5.13. Incrementando la pre-sión del agua en el interior del globo esférico, al cual se le han practicado orificios en distintos lugares, se puede apreciar cómo los chorros de agua salen siempre en dirección radial, es decir, perpendiculares a las paredes del globo.

La fuerza que un fluido ejerce sobre una superficie tiene su origen en la colisión de las moléculas del fluido contra la superficie.

De acuerdo a la 3a ley de Newton y la relación entre impulso y momentum lineal, cada colisión produce una fuerza sobre la superficie.

Como el número de moléculas del fluido es muy grande, también se produce un gran número de colisiones con las super-ficies en cada instante, lo que da lugar a la fuerza macroscópica constante que relacionamos con la presión.

Figura 5.12. (a) Las fuerzas del líquido sobre cada punto en las superficies del objeto apuntan en todas direcciones (flechas azules y verdes, por ejemplo), pero una parte de ellas se cancela mutuamente y solo resulta una fuerza neta perpendicular a la superficie (flecha roja). (b) Por lo tanto, la fuerza que ejerce un líquido sobre cualquier punto de un objeto sumergido es perpendicular a la superficie del objeto. La misma idea se puede aplicar a las paredes del recipiente: la fuerza que ejerce el líquido sobre las paredes es perpendicular a ellas en todos los puntos.

(a) (b)

Page 16: FLUIDOS - FISICA

127

Capítulo 3: Hidrostática

Cuando nadamos con la cabeza bajo el agua, podemos notar cómo la presión aumenta en la medida en que más nos sumergimos. A veces, llegamos a sentir un pequeño dolor en el oído, producto del aumento de presión sobre el tímpano. ¿Qué provoca esa presión?

Simplemente, el peso del fluido que está sobre nosotros. ¿Y qué fluido tenemos sobre nosotros? Agua, obviamente. Pero no solo agua. Sobre la superficie del agua hay aire, y como tal, también es un fluido cuyo peso tenemos que considerar.

El peso del aire de la atmósfera produce una presión sobre la su-perficie terrestre y sobre cualquier otra superficie que se encuentre en ella; por ejemplo, la superficie de nuestros cuerpos. Esta presión es llamada presión atmosférica.

Analicemos la relación entre el peso del agua, el peso del aire y la presión, cuando un objeto se sumerge a cierta profundidad.

Para esto, consideremos el diagrama de la Figura 5.14, en el que se ha representado una porción del agua en reposo, como un cilindro de masa m, cuyo volumen es:

V = A · h (5.4)

Donde A es el área de la base circular del cilindro y h es la pro-fundidad de la columna de líquido.

Como el cilindro de agua está en reposo, todas las fuerzas que actúan sobre él están en equilibrio. Por una parte, actúa la fuerza de gravedad, que identificamos como el peso de la porción de agua contenida en el cilindro imaginario (

FC ). Por otra parte,

actúa también el peso de la columna de aire atmosférico que se encuentra justo arriba del cilindro de agua (

Fatm ). Ambas fuerzas

actúan en dirección vertical y hacia abajo, de modo que hay una fuerza que las equilibra apuntando en sentido contrario. Esta fuerza la identificamos como la fuerza que ejerce el resto del fluido sobre el cilindro de agua (

FFluido ).

Aunque el cilindro de agua que estamos imaginando no es un cuerpo rígido, por un momento supongamos que las tres fuerzas en equilibrio actúan en el centro del cilindro. De acuerdo a esto, considerando los módulos de las fuerzas, el equilibrio que hemos mencionado se puede escribir del siguiente modo:

F F FFluido C atm= + (5.5)

Figura 5.14. En el esquema, se repre-senta un cilindro imaginario que contiene un porción del agua del recipiente, y cuya masa es m. El peso del cilindro de agua ( FC

)ejerce una presión sobre

el resto del fluido en su base, a una profundidad h. Como el líquido está en reposo, las fuerzas que actúan sobre el agua están en equilibrio. Además del peso del agua, actúa sobre la cara superior del cilindro el peso de la columna de aire atmosférico (Fatm

),

inmediatamente arriba de él. Ambas fuerzas son equilibradas por la fuerza opuesta (FFluido

) que ejerce el resto del

líquido sobre la porción contenida en el cilindro imaginario.

Sección 5: Presión y principio de Pascal

atmósfera

A

h

FC

FFluido

Fatm

En estricto rigor, el peso del aire atmos-férico actúa sobre la cara superior del cilindro, mientras que la fuerza del resto del fluido actúa sobre la base inferior del cilindro. La diferencia entre estas fuerzas equivale al peso del fluido contenido en el cilindro.

Page 17: FLUIDOS - FISICA

Física 3° Año Medio128

Fluidos

Para obtener el equilibrio de fuerzas expresado en la ecuación (5.5), hicimos uso de una suposición correcta, pero imprecisa. Es correcta, porque si ese equilibrio no se produjera, entonces la masa de agua contenida en el cilindro que hemos considerado se desplazaría, lo que contradice la idea de que se trata de un fluido en reposo.

Sin embargo, se trata de una suposición imprecisa, porque las fuerzas no actúan en el centro del cilindro de agua, sino que lo hacen de manera distribuida, en el área de la base del cilindro, a una profundidad h. Es decir, el concepto apropiado para establecer el equilibrio de la ecuación (5.5) es el de presión.

Ya sabemos que para obtener la presión ejercida por una fuerza, es necesario dividir la intensidad de la fuerza por el área en la que actúa. Es decir, dividiendo por A todos los términos de la ecuación (5.5), obtenemos:

F F FF

AFA

FA

Fluido C atm

Fluido C atm

= +

= + (5.6)

Considerando lo anterior, podemos reescribir la ecuación (5.6) del siguiente modo:

FA

FA

FA

Pm g

AP

Fluido C atm= +

=⋅

+ 0 (5.7)

Donde hemos considerado que FC es el peso del cilindro de agua. Haciendo uso de las relaciones (5.1) y (5.4), la ecuación anterior queda como:

P

V gA

P

PA h g

AP

=⋅ ⋅

+

=⋅ ⋅ ⋅

+

ρ

ρ

0

0

(5.8)

Es decir, encontramos que la presión del fluido a una profundidad h es:

P h g P= ⋅ ⋅ +ρ 0 (5.9)

Figura 5.15. Para medir la presión atmosférica, (a) Torricelli llenó un tubo con mercurio, cerrado en uno de sus extremos, y luego lo tapó con el dedo. (b) A continuación, invirtió el tubo, lo sumergió en una cubeta con mercurio y retiró el dedo cuidando que no entrara aire en el tubo. El mercurio en el tubo descendió hasta una altura de 76 cm aproximadamente, mientras en la parte superior se formó un vacío parcial. El tubo de mercurio no se vacía porque el aire de la atmósfera ejerce una presión sobre la superficie del mercurio en la cubeta, suficiente para equilibrar el peso de la columna de mercurio de 76 cm.

A B

Page 18: FLUIDOS - FISICA

129

Capítulo 3: Hidrostática

En sistemas alternativos de unidades, la presión se puede medir en: Torricelli (Torr), milímetros de mercurio (mmHg), atmósferas (atm), libras por pulgada cuadrada (psi), bar (bar), entre otras.

Algunas equivalencias entre estas uni-dades, referidas al valor de la presión atmosférica a nivel del mar, son las siguientes:

1 14 71 1013 101 760

1

2

atm psiatm Paatm Torr

Tor

== ⋅=

,

rr mmHg= 1

Figura 5.16. Cuando se nada a ma-yor profundidad, mayor es la presión experimentada.

En esta expresión, el primer término del lado derecho es la pre-sión del fluido sobre la base del cilindro de agua, es decir, es la presión a una profundidad h, conocida como presión hidrostáti-ca. Por su parte, el segundo término es la presión que ejerce el aire de la atmósfera sobre el cilindro o, simplemente, la presión atmosférica (P0).

Por lo tanto, este resultado muestra que la presión del fluido de-pende directamente de la profundidad y de la densidad del fluido y también de la presión en su superficie.

Además, de la ecuación se deduce que la presión es la misma en todos los puntos del fluido situados a la misma profundidad, inde-pendientemente de la forma del recipiente que lo contiene.

La presión atmosférica a nivel del mar es de 1013 · 102 Pa. En adelante, aproximaremos este valor como P0 = 105 Pa

Ejemplo 3

Una persona se encuentra sumergida a una profundidad de 3 m en una piscina de agua.

a) ¿Qué presión ejerce el agua sobre la persona?

b) Si el área de cada uno de sus tímpanos es de 1 cm2, ¿qué fuerza soportan?

a: De acuerdo a la ecuación (5.9), para encontrar la presión hidrostática sobre la persona, consideramos la densidad del fluido, la profundidad y la presión atmosférica. Usando g

ms

= 102

y considerando P0 a nivel del mar, encontramos:

b: Para encontrar la fuerza que soporta el tímpano usamos la definición operacional de presión dada por la ecuación (5.2):

F = P · A

P h g P

Pkgm

mms

Pa

P

o= ⋅ ⋅ +

= ⋅ ⋅ +

=

ρ

1 000 3 10 10

30 00

3 25

00 10

0 3 10 101 3 10

25

5 5

5

kgm s

Pa

P Pa PaP Pa

⋅+

= ⋅ += ⋅

,,

Sección 5: Presión y principio de Pascal

Page 19: FLUIDOS - FISICA

Física 3° Año Medio130

Fluidos

Antes de reemplazar los datos del enunciado, tenemos que considerar las unidades apropiadas. En particular, para el área de cada tímpano, tenemos:

1 1 10

1 10

2 2 2

2 4 2

cm m

cm m

= ( )=

Entonces,F P AF Pa mF N

= ⋅= ⋅ ⋅=

−1 3 10 1013

5 4 2,

Este resultado muestra que, a 3 m de profundidad, el tímpano resiste una fuerza relativamente grande.

En la Figura 5.18, todos los agujeros laterales del reci-piente tienen el mismo diámetro. ¿A qué se debe que el chorro de agua tenga mayor o menor alcance horizontal al salir del recipiente?

Principio de PascalEn 1648, Blaise Pascal descubrió, realizando experimentos con fluidos, lo siguiente:

«El incremento de presión aplicado a la superficie de un fluido incompresible, contenido en un recipiente indeformable, se trans-mite con el mismo valor a cada una de las partes del mismo»

Este enunciado se conoce como principio de Pascal.

El montaje que se muestra en la Figura 5.13 también es una de-mostración del principio de Pascal, ya que la presión ejercida por la jeringa se propaga de manera constante a cualquier lugar en el interior del líquido, lo que queda en evidencia porque se observa que el agua sale por todos los agujeros con la misma presión.

El principio de Pascal es utilizado en muchos objetos tecnológicos que trabajan con líquidos. Por esta razón, estas máquinas se llaman hidráulicas, ya que usan los fluidos para aplicar y aumentar las fuerzas.

Piensa, por ejemplo, en los componentes de un vehículo: ¿qué características tienen en común la dirección hidráu-lica, los frenos hidráulicos y la gata hidráulica?

Figura 5.17. ¿En cuál de los recipien-tes la presión del líquido en el fondo es mayor?

Figura 5.18. Debido a que la presión es perpendicular a las paredes del recipiente, los chorros de agua salen inicialmente en esa dirección y luego se curvan por efecto de la fuerza de gravedad. Los tres orificios tienen el mismo diámetro.

Page 20: FLUIDOS - FISICA

131

Capítulo 3: Hidrostática

A continuación, analizaremos el interesante caso de la gata hidráulica, que consiste en un dispositivo capaz de levantar un gran peso a partir de la aplicación de una fuerza relativamen- te pequeña.

Como se muestra en la Figura 5.19, el mecanismo de la gata hidráulica está compuesto por dos émbolos de distinto diámetro conectados por un fluido encerrado en una cavidad, cuyo diámetro varía de un émbolo al otro.

Al mecanismo se aplica fuerza de entrada (F1) sobre una pequeña superficie de área A1. Esto genera una presión en el fluido que se transmite de manera constante en todo su interior y, en particular, hasta la superficie A2, cuya área es mayor que A1. Por lo tanto, sobre A2 el fluido aplica una fuerza de salida (F2) que es mayor que la fuerza de entrada.

Figura 5.20. Con un par de jeringas de distinto diámetro, una pequeña man-guera y un líquido, se puede demostrar fácilmente el principio de Pascal.

Figura 5.19. Por el principio de Pascal, la fuerza aplicada sobre el ém-bolo 1 es amplificada gracias a que la presión ejercida en el fluido es constante.

La fuerza aplicada sobre el émbolo 1 provoca una presión (P1) extra sobre el fluido, que se transmite en todo su interior; en particular, hasta el émbolo 2. Por lo tanto, por el principio de Pascal:

P1 = P2 (5.10)

Donde P2 es la presión extra sobre el émbolo 2.

A continuación, haciendo uso de la ecuación (5.2), podemos escribir las presiones en términos de fuerza y área.

F1

F2

A1

1

2

A2

Sección 5: Presión y principio de Pascal

Page 21: FLUIDOS - FISICA

Física 3° Año Medio132

Fluidos

Figura 5.21. Típica gata hidráulica para levantar un automóvil mediano. ¿Dónde está el émbolo de entrada y el émbolo de salida?

Es decir:

FA

FA

FA

A F

FAA

F

1

1

2

2

1

12 2

12

12

=

⋅ =

⋅ =

(5.11)

Este resultado muestra claramente que el factor de aumento del área en el émbolo 2 determina un aumento proporcional de la fuerza de salida. Es decir, cuando mayor es el área de salida, en comparación con el área de entrada, mayor es la fuerza útil o de carga de la máquina hidráulica.

Ejemplo 4

Consideremos el mecanismo de una gata hidráulica en la cual la fuerza de entrada es de 100 N y se aplica sobre un área de 100 cm2. El área de la superficie de salida es de 10 000 cm2.

a) ¿Cuál es la fuerza de salida en este caso?

b) ¿Es suficiente la fuerza de salida para levantar un automóvil de 1 500 kg?

a: Por el principio de Pascal, la presión ejercida por la fuerza de entrada es la misma que se ejerce sobre la superficie de salida. De acuerdo a la última de las ecuaciones (5.11), tenemos:

FAA

F

Ncm

cmF

N F

12

12

2

2 2

2

10010 000

10010 000

⋅ =

⋅ =

=

Es decir, la fuerza aumentó 100 veces en relación a la fuerza aplicada.

b: Como el peso de un automóvil de 1 500 kg es aproxima-damente 15 000 N, la fuerza de salida de la gata hidráulica del ejemplo no es suficiente para levantarlo.

Hay varias formas de modificar el funcionamiento de la gata para lograr que levante el vehículo, ¿cuáles son?

Figura 5.22. El sistema de frenos de un vehículo también utiliza el principio de Pascal, ya que mediante un fluido se transmite la presión ejercida por la fuerza en el pedal hasta la balata, que con una fuerza mayor presiona el tambor del neumático para frenarlo.

pedal

Caja maestra

neumáticotambor

balatamanguera

Depósito de líquido de frenos

Page 22: FLUIDOS - FISICA

133

Capítulo 3: Hidrostática

Figura 5.23. En una gata hidráulica, aprovechando su peso, un hombre de 75 kg logra levantar un auto de 850 kg. Es interesante observar que lo que se desplaza el émbolo hacia abajo es considerablemente mayor que lo que se desplaza el automóvil hacia arriba.

Analizando la situación representada en la Figura 5.23, ¿cómo se relaciona el principio de Pascal con el trabajo y la energía mecánica?

Presión atmosféricaYa hemos mostrado que los gases, a diferencia de los líquidos, pueden ser comprimidos. Nuestra atmósfera es un fluido gaseoso en el que la densidad disminuye gradualmente con la altitud.

Entre las capas atmosféricas, la que se encuentra más próxima a la superficie del planeta es llamada troposfera, y tiene la mayor densidad, porque está más comprimida por el peso de las capas superiores.

De esta manera, en la medida que nos alejamos de la superficie de la Tierra la densidad disminuye.

De acuerdo a esto, la atmósfera puede ser modelada como un fluido estático formado por capas de distinta densidad. Si en este modelo se considera, además, que la temperatura y la intensidad del campo gravitatorio son constantes, entonces la densidad atmosférica es directamente proporcional a la presión.

Al formalizar matemáticamente estas condiciones, la presión atmosférica muestra una relación exponencial con la altitud. Es decir, la presión atmosférica disminuye rápidamente al alejarse de la superficie terrestre.

Figura 5.24. El 19 de septiembre de 1648, un año después de recibir una carta desde París de parte de Blaise Pascal, su cuñado, Florin Périer junto a un grupo de amigos, siguiendo las instrucciones indicadas en esa carta, realizaron el experimento de Torricelli en la cima del Puy de Dôme, en la región central de Francia. Tal como había comprendido Pascal que sucedería, la altura de la columna de mercurio en el barómetro fue 85 mm menor que en la base de la montaña, aproximadamente 1 000 m más abajo.

Sección 5: Presión y principio de Pascal

Page 23: FLUIDOS - FISICA

Física 3° Año Medio134

Fluidos

Se puede demostrar, dadas las condiciones anteriores, que la pre-sión atmosférica depende de la altura sobre el nivel del mar (h) de la siguiente forma:

P P eo

hkm= ⋅

−8 55, (5.12)

Donde Po es la presión atmosférica a nivel del mar. Esta expresión es una buena aproximación para la presión atmosférica a alturas relativamente bajas.

La función exponencial es una función matemática muy importante en innu-merables procesos naturales y se puede escribir como:

f x ex( ) = (5.13)

Donde el número e corresponde a un irracional, cuyas primeras cifras de-cimales son 2,7182818284. Algunos procesos gobernados por la función exponencial son: el número de células de un feto mientras se desarrolla en el útero materno, el número de bacterias que se reproducen por mitosis o el número de contagiados en una epidemia de gripe, entre otros.

Figura 5.26. Manómetro de tubo de vidrio, que indica la diferencia de pre-sión entre dos fluidos gaseosos. Para medir la diferencia, basta determinar la diferencia de altura de la columna de mercurio entre los dos tubos paralelos. Si cada rama del manómetro se conecta a distintas fuentes de presión, el nivel del líquido aumenta en la rama a menor presión y disminuye en la otra.

Para medir la presión absoluta (Pabs) de un gas, a la presión manométrica (Pman) se debe sumar la presión atmosférica (P0). La presión manométrica típica de un neumático de bicicleta, por ejemplo, es de 300 a 450 kPa:

Pabs = Pman + P0 (5.14)

Figura 5.25. Modelo para la variación de la presión atmosférica con la altitud sobre el nivel del mar. Se observa que la tropósfera tiene la mayor densidad, porque está más comprimida por el peso de las capas superiores de aire.

A pesar de lo anterior, es evidente que la atmósfera no puede ser considerada realmente como un fluido estático, ya que hay una serie de factores que hacen de ella un sistema dinámico. Por ejemplo:

• Las diferencias de temperatura entre masas de aire polar y masas de aire proveniente de los trópicos, cuya interacción produce los denominados frentes meteorológicos.

• La diferencia de temperatura entre el mar y las montañas, que generan vientos locales.

• La rotación del planeta, que produce el efecto Coriolis sobre las masas de aire que se desplazan siguiendo un meridiano.

• Las diferencias de temperatura entre masas de aire a diferentes altitudes, que producen zonas de ascenso y descenso de aire, los llamados ciclones y anticiclones.

Tropo-

pausa

Tropopausa

Troposfera

Estratosfera

moléculas de aire

Everest

Alti

tud

(km

)40

30

20

10

00,1 0,3 0,5 0,7 1océano

900

800

700

600

500

400

300

200

100

0 mm

Gas 1 Gas 2Presión amosférica (atm)

Page 24: FLUIDOS - FISICA

135

Capítulo 3: Hidrostática

A partir de la lista de conceptos relevantes (CR) y frases conectoras (FC), completa en tu cuaderno el mapa conceptual de la figura.

Conceptos Relevantes (CR) Frases Conectoras (FC)A Aceleración de gravedad I Es producto deB Líquidos II Es cuociente entreC Gases III Aplica el concepto deD Principio de Pascal IV Depende deE Presión atmosférica V Estudio deF Densidad de fluidoG Masa

Síntesis

Presión 4

Hidrostática

DesafíoCuando hayas terminado esta

actividad, vuelve a leer el texto de la sección, con mucha aten-ción, y genera tu propio mapa

conceptual.

5

En

Reposo

Cumplen el

7

Se relaciona con

Presión hidrostática 6

Es producida por el

Peso del aire atmosférico

12

1

Profundidad

3

11

2

Volumen

9

Fuerza aplicada

Área de contacto

Sección 5: Presión y principio de Pascal

8 10

Si son compresibles

Page 25: FLUIDOS - FISICA

Física 3° Año Medio136

Fluidos

Preguntas y ejercicios propuestos

1 ¿Qué quería demostrar Pascal con el experimento que le encargó realizar a su cuñado en el monte Puy de Dôme?

2 ¿Cuál se cree actualmente que es el estado de la materia más abundante en el Universo?

3 ¿Por qué duele más un pinchazo de una aguja, aunque sea con una fuerza muy leve, que un empujón de gran fuerza hecho con la mano ex-tendida?

4 ¿Cómo se relaciona el principio de Pascal con el trabajo?

5 Una caja cúbica de madera que posee una masa de 50 kg y aristas de 1 m de longitud, se ubica en el piso liso de la sala. (a) ¿Cuál es el área de una de sus caras? (b) ¿Cuál es el peso de la caja? (c) ¿Cuál es la presión que ejerce la caja sobre el suelo? (d) ¿Cuál es presión de la caja sobre el suelo, si un niño de 50 kg se sube sobre ella?

6 Un ladrillo de 5 kg de masa tiene las siguientes dimensiones: 20 cm de ancho, 40 cm de largo y 10 cm de espesor. (a) ¿Cuál es el área de la cara de mayor superficie? (b) ¿Cuál es el volumen del ladrillo? (c) ¿Cuál es valor de su densidad? (d) ¿Cuál es el módulo del peso del ladrillo? (e) ¿Qué presión ejerce el peso del ladrillo sobre el piso a través de su área mayor?

7 En su casa, un joven recibe el encargo de apilar 12 ladrillos en una habitación, pero se le indica que no agrupe una sola columna vertical, porque el piso soporta solo la cuarta parte de la presión que esa distribución produce. (a) ¿Cuál sería la distribución más eficiente; es decir, la que ocu-paría menos área, pero evitando que se rompa el piso? Explica.

8 En un laboratorio de fluidos se tiene tres pisci-nas de 15 m de profundidad cada una, abiertas

a la atmosfera: una de aceite, una de agua pura y una de mercurio. ¿A qué profundidad, en cada piscina, la presión absoluta es de: (a) 2Po (b) 3Po? (c) ¿A qué profundidad, en cada caso, el

líquido ejerce una presión equivalente a 1 atm (Po)?

(Considere para ambas respuestas g = 9,8 m/s2)

( ρaceitekgm

= ⋅0 91 1033, ; ρagua

kgm

= ⋅1 1033 ;

ρmercuriokgm

= ⋅13 6 1033, )

9 El último piso de un edificio está a 50 m del suelo. Si el sistema de agua potable tiene una presión absoluta de 4 · 105 Pa, (a) ¿hasta qué altura sube el agua en estas condiciones? (b) ¿Es necesario instalar una bomba para elevar el agua hasta el último piso?

10 Observa la Figura 5.27. Una persona de 80 kg de masa se sube sobre la plataforma, la que tiene un radio de 20 cm. (a) ¿Cuál es el área de la sección transversal de la plataforma cilíndrica? (b) Si el fluido es agua pura, ¿qué altura (h) alcanza en el tubo abierto a la atmósfera?

Figura 5.27

11 Se suele decir que la pisada de un elefante no la soporta ningún ser vivo. Supongamos que un joven elefante tiene una masa de 40 toneladas y que se para equilibradamente sobre sus cuatro patas. Si la superficie de apoyo de cada pata se puede modelar como un círculo de 30 cm de diámetro, (a) ¿cuál es el área de cada pata? (b) ¿Cuál es el

Po

h

Po

Page 26: FLUIDOS - FISICA

137

Capítulo 3: Hidrostática

módulo del peso del elefante? (c) ¿Qué presión ejerce cada pata del elefante sobre el suelo?

12 Una mujer de 60 kg, de pie en la fila de un banco, usa zapatos con tacos delgados. El área de la suela más el área de la tapilla del taco, de cada zapato, suma 6 cm2. (a) ¿Qué presión ejerce uno de sus pies sobre el suelo, al estar normalmente de pie? (b) ¿Cuánto se reduce la presión anterior si cambia los zapatos por zapatillas, cuya superficie de apoyo es 10 veces mayor?

Figura 5.28

13 Un tubo de 80 cm de largo mantiene en posición vertical y se llena hasta la mitad con mercurio y la otra mitad con agua. (a) ¿Cuál es la presión manométrica en el punto de contacto de los lí-quidos? (b) ¿Cuál es la presión manométrica en el fondo del tubo?

14 En la Figura 5.29, se muestra un sistema mecánico en equilibrio en el cual el diámetro del pistón de entrada es de 25 cm y el de salida de 75 cm. La persona que ejerce la fuerza en la entrada tiene una masa de 70 kg. (a) ¿Cuál es el módulo del peso de la persona? (b) ¿Qué presión ejerce el émbolo de entrada cuando la persona se para sobre él? (c) ¿Cuál es el peso del vehículo?

Figura 5.29

15 En la Figura 5.30, se muestra un tubo en forma de U que contiene dos volúmenes iguales de agua y mercurio. Cada líquido ocupa una extensión de 20 cm del tubo. (a) ¿Cuál es la presión manomé-trica en la sección transversal donde se tocan los líquidos? (b) ¿Cuál es la diferencia de altura (ya – yb) entre los líquidos?

Figura 5.30

16 De acuerdo a la Figura 5.31, sobre la jeringa pequeña, de radio ro, se ejerce una fuerza Fo que genera una presión sobre el líquido contenido en la manguera. La presión se transmite a través del fluido líquido en el sistema hasta otra jeringa más grande, cuyo émbolo tiene un radio de igual a 2ro. (a) ¿Cuánto aumenta la fuerza del sistema en la jeringa grande? (b) Si el émbolo de la jeringa pequeña se comprime 3 cm, ¿cuál es el trabajo realizado? (c) En estas condiciones, ¿cuánto se desplaza el émbolo de la jeringa grande?

Figura 5.31

ym

ya

agua

Sección 5: Presión y principio de Pascal

mercurio

Page 27: FLUIDOS - FISICA

Física 3° Año Medio138

Fluidos

Evaluación final de la sección

PARTE I: Anota en el recuadro el número de la magnitud que corresponde a la unidad.

Magnitud Unidad1 Profundidad Kg/m3

2 Presión hidrostática Pa3 Área m2

4 Densidad m

PARTE II: Indica si el enunciado es verdadero o falso. Expresa en tu cuaderno la justificación de tus respuestas.

V o F1 La presión al interior de un fluido incompresible es constante.2 La presión atmosférica aumenta linealmente con la altura, respecto al nivel del mar.3 La presión externa aplicada a un fluido incompresible en reposo se reduce en los puntos más

alejados del fluido.4 La presión de un fluido solo depende de su densidad.5 La presión atmosférica es producto de la fuerza que ejercen los vientos.

PARTE III: Responde las siguientes preguntas, marcando la alternativa correcta.1 Una fuerza F ejerce una presión P, sobre un

área circular. Si el radio del círculo aumenta hasta un valor que es 2 veces el radio original y la presión se mantiene constante, ¿cuánto aumentó la fuerza?a) 2 veces. b) 3 veces. c) 4 veces.d) Falta información.

2 Se construyen dos barómetros de mercurio, omo el de Torricelli, usando dos tubos de distinto diámetro. ¿En cuál de los barómetros a columna de mercurio es más alta?a) En el de mayor diámetro.b) En el de menor diámetro.c) En ninguno, ambos quedan en equilibrio a

la misma altura.d) Falta información.

3 Los radios de los émbolos de una gata hidráulica son de 2 cm y 20 cm, respectivamente. ¿Qué presión ejerce el émbolo mayor para levantar un automóvil, si sobre el émbolo menor actúa una presión de 5·105 Pa?a) 5·105 Pab) 0,25·105 Pa c) 200 π Pad) 5 Pa

4 ¿Cuál de las siguientes opciones no corresponde a la presión atmosférica a nivel del mar?a) 14,7 Psib) 105 Pac) 760 Torrd) 760 cmHg

Page 28: FLUIDOS - FISICA

139

Capítulo 3: Hidrostática

Indagación N°16¿Cómo flotan los barcos?

PARTE I. Trabajo personal

Con seguridad has observado embarcaciones pe-queñas y otras gigantescas que navegan en el mar. Sin duda, el caso más desconcertante es el de las naves de gran tamaño y peso, como en el caso de los trasatlánticos, que son verdaderos edificios flotantes. ¿Qué pasaría si tomamos todo el metal y los otros materiales que componen un barco, luego hiciéra-mos una esfera homogénea con ellos e intentáramos ponerla en flotación? ¿Se hundiría?

a) Lo anterior, se puede modelar con un trozo de plasticina. ¿Podría flotar una esfera de plasticina en el agua?

b) ¿Qué magnitud física es necesario cambiar para que la esfera de plasticina flote?

PARTE II. Trabajo en equipo

Junto a un compañero o una compañera, contrasten sus respuestas y argumenten a favor o en contra de ellas.

A continuación, elaboren una hipótesis en conjunto que dé respuesta a la segunda pregunta.

a) Registren la hipótesis en sus cuadernos e identifiquen cuáles son las variables observables que pueden medir y/o controlar.

b) Una vez planteada su hipotesis, diseñen un procedimiento experimental que les permita ponerla a prueba, para evaluar si es una explicación aceptable o debe ser descartada. Dibujen su montaje experimen-tal y describan brevemente, pero con precisión, el procedimiento que sugieren.

Procuren que el procedimiento experimental propuesto sea factible de realizar en una hora de clases; es decir, que incluya el uso de materiales de fácil adquisición o construcción y tiempos razonables para la obser-vación y el análisis de sus resultados.

c) Para finalizar, elaboren un informe de dos páginas según las indica-ciones que les dé su profesor o profesora.

Recuerden que una hipótesis es una explicación posible que se supone cierta hasta que pueda ser contrastada empíricamente. Por esta razón, es fundamental que la hipótesis se refiera a un número reducido de variables observables y de algún modo medibles, que eventualmente pueden ser controladas en un experimento.

Imagen 6.1

Page 29: FLUIDOS - FISICA

Física 3° Año Medio140

Fluidos

El principio de Arquímedes

Arquímedes de Siracusa vivió entre los años 287 y 212 A.C. Entre sus descubrimientos más notables está el principio de flotabilidad de los cuerpos, conocido hoy como principio de Arquímedes.

Arquímedes descubrió que un cuerpo, al ser sumergido parcial o totalmente en el interior de un fluido, experimenta una fuerza hacia arriba, llamada fuerza de empuje o, simplemente, empuje, cuyo módulo es igual al peso del fluido que desplaza.

Figura 6.2. El peso de un objeto flotante es igual al peso del agua que desplaza su parte sumergida. Este es el principio de Arquímedes.

6

Figura 6.1. El aumento del nivel de agua en el jarro es el mismo que se tendría si, en vez de poner la piedra en el jarro, se vertiera en él un volumen de agua igual al volumen de la piedra.

En términos de módulos, el empuje se define, entonces, del si-guiente modo:

E Pfd= (6.1)

Donde E es la fuerza de empuje y Pfd corresponde al peso del fluido desplazado.

Es importante no confundir el peso del fluido desplazado con el peso del objeto sumergido. El primero depende de la masa del fluido desplazado (mfd):

P m gfd fd= ⋅ (6.2)

líquido desplazado

Sección

Page 30: FLUIDOS - FISICA

141

Capítulo 3: Hidrostática

Como sabemos, el peso del objeto, en cambio, es:

P = m · g (6.3)

Ya que el fluido desplazado es el líquido que sube en el contenedor cuando se introduce parcial o totalmente un objeto, ¿cómo podrías determinar el peso del fluido desplazado?

Ejemplo 5

Una bolita de acero se introduce en un vaso de precipitado que contiene agua pura. Una vez que la bolita está dentro del líquido se saca con una pipeta exactamente la cantidad de agua desplazada por el objeto, es decir, el recipiente vuelve a tener el nivel de líquido inicial. Al medir la masa del agua extraída, se obtienen 10 g.

a) ¿Cuál es el peso del agua desplazada?

b) ¿Cuál es el módulo del empuje que experimenta la bolita de acero?

a: Para encontrar el peso del agua desplazada solo necesi-tamos conocer su masa. De acuerdo a la ecuación (6.2), tenemos:

P m g

P kgms

P N

fd fd

fd

fd

= ⋅

= ⋅ ⋅

=

−10 10 10

0 1

32

,

b: De acuerdo al principio de Arquímedes, expresado en la ecuación (6.1), obtenemos:

E P

E Nfd=

= 0 1,

En ocasiones, se conocen las densidades del fluido y del objeto, así como el volumen de este cuerpo. Por eso, el principio de Arquímedes también se puede aplicar considerando el concepto de densidad.

En general, la densidad del fluido ( ρ ) es diferente de la densidad del objeto ( ρ0 ). Veremos a continuación que la relación entre estas cantidades determina la flotación del cuerpo.

Figura 6.3. El principio de Arquímedes se aplica al comportamiento de los fluidos en general. Así, un globo ae-rostático asciende cuando su peso es menor que el peso del aire atmosférico que desplaza.

E

P

Sección 6: Principio de Arquímedes

Page 31: FLUIDOS - FISICA

Física 3° Año Medio142

Fluidos

¿Por qué un objeto se hunde o flota?La flotación de un objeto depende de la relación entre su densidad y la densidad del fluido en el que se encuentra. Analizaremos los tres casos posibles.

El objeto es más denso que el fluido

En este caso, el objeto se va hacia el fondo del líquido en el que es sumergido, debido a que el peso del objeto es mayor que el peso del fluido desplazado y, por lo tanto, mayor que el empuje:

P > E (6.4)

La piedra sumergida completamente en la Figura 6.1 es un buen ejemplo de esta situación.

El objeto tiene la misma densidad que el fluido

En este caso, no podemos decir que el objeto se hunda o flote, aunque se trata de un caso particular en el que el peso del objeto es igual al peso del fluido desplazado y, por lo tanto, igual al empuje. Sin embargo, el objeto podría encontrarse igualmente en el límite de la superficie del fluido o en el fondo.

P = E (6.5)

Un ejemplo de esta condición sería la situación de un globo lleno de agua en el interior de otro recipiente con agua.

Figura 6.5. En muchos peces, la vejiga natatoria permite controlar la flotabili-dad mediante un complejo sistema de intercambio gaseoso con la sangre. El mecanismo permite al pez ascender o descender en el agua, cambiando la densidad relativa del pez sin necesidad de utilizar la musculatura.

Figura 6.4. Un globo lleno de agua sumergido en una piscina se encuen-tra en una situación en la que su peso está completamente equilibrado por el empuje, y por esta razón no flota, pero tampoco se hunde hasta el fondo.

Page 32: FLUIDOS - FISICA

143

Capítulo 3: Hidrostática

El objeto tiene menor densidad que el fluido

En este caso el objeto permanece parcialmente sumergido, es decir, flota. Esto se debe a que si el cuerpo se sumerge completamente, su peso es menor que el peso del fluido que desplaza, de manera que asciende hasta la superficie.

En estas condiciones, el objeto flotante desplaza un volumen de agua que es una fracción del volumen total del objeto, lo que permite equilibrar su peso y el empuje.

Por supuesto, los ejemplos de esta situación son numerosos. Tal vez, el más espectacular sea el de un iceberg en el mar, cuya versión do-méstica podemos observar con cubos de hielo en un vaso de agua.

Figura 6.6. Un objeto cuya densidad neta es menor que la del agua desplaza un volumen de agua que es una fracción del volumen total del objeto.

¿Qué le ocurre a un fluido como el aceite si se introduce en agua? ¿Sube a la superficie del agua o baja a lo más profundo? ¿Por qué?

En suma, el principio de Arquímedes se puede expresar en función de la densidad del fluido del siguiente modo:

E P

E m g

E V g

fd

fd

fd

=

= ⋅

= ⋅ ⋅ρ

(6.6)

Figura 6.7. El hielo flota porque su densidad es menor que la densidad del agua líquida.

Figura 6.8. Un submarino utiliza el principio de Arquímedes para navegar bajo el agua o en la superficie. Para controlar su peso, los submarinos es-tán equipados con tanques de lastre. Para sumergirse o emerger, usan los tanques de proa y popa, llamados tanques principales, que se abren y se llenan completamente de agua para sumergirse o se llenan de aire a presión para emerger.

Sección 6: Principio de Arquímedes

Page 33: FLUIDOS - FISICA

Física 3° Año Medio144

Fluidos

Así, para un objeto flotante, la condición de equilibrio en función de su densidad (ρ0) y la densidad de fluido (ρ) es:

P Em g V g

V Vfd

fd

=⋅ = ⋅ ⋅

⋅ = ⋅

ρ

ρ ρ0

(6.7)

Ejemplo 6

Un iceberg, como el de la Figura 6.6, tiene una densidad de 920 kg/m3 y flota en la superficie del agua de mar, cuya densidad es de 1 030 kg/m3.

a) ¿Qué fracción del iceberg se encuentra sobre la superficie del mar?

a: Un objeto flotante experimenta un empuje igual a su peso, ya que está en equilibrio en la superficie; por lo tanto, de acuerdo al desarrollo de las ecuaciones (6.7), tenemos:

P Em g V g

V V

V V

kgm

fd

fd

fd

=⋅ = ⋅ ⋅

⋅ = ⋅

⋅ =

ρ

ρ ρ

ρρ

0

0

3920

10300

0 89

3

kgm

V V

V V

fd

fd

⋅ =

⋅ =,

El equilibrio de fuerzas consiste en que el peso del iceberg es igual al peso del agua desplazada, lo que se logra cuando una gran parte del iceberg está sumergida. Esta porción tiene un volumen igual al volumen del agua desplazada.

Por lo tanto, solo el 11% del volumen del iceberg es visible sobre la superficie.

¿Qué fuerzas actúan sobre un objeto sumergido, cuya densidad es mayor que la del fluido en el que se encuentra? Realiza un diagrama de cuerpo libre para ilustrarlas.

Figura 6.9. En (a), el dinamómetro mide el peso del objeto. En (b), cuando se sumerge el objeto en un fluido, el dinamómetro mide un peso menor, que se conoce como peso aparente. En este caso, el dinamómetro marca menos debido a que al peso del objeto se le resta la fuerza de empuje ejercida por el agua. Este es un método directo para medir el empuje.

(a) (b)

P Pap

Page 34: FLUIDOS - FISICA

145

Capítulo 3: Hidrostática

Actividad de profundización

¿Cómo se relaciona el peso de un objeto con la fuerza de empuje que experimenta en un fluido?

Para realizar esta actividad, se necesita lo siguiente: una botella de plástico de 2,5 litros, agua y un tubo de ensayo.

Reúnete con algunos compañeros y compañeras (entre 2 y 5, idealmente) y formen un equipo de trabajo.

a) Reflexionen sobre la siguiente pregunta: ¿cómo se relacióna el peso de un objeto que se sumerge en agua con la fuerza que el agua ejerce sobre él? Planteen como equipo varias hipótesis y elijan luego, por consenso, la que estimen que es una mejor explicación.

A continuación, realicen el siguiente experimento:

Llenen la botella con agua hasta el tope, sin dejar aire en el cuello de la botella. Luego viertan agua al tubo de ensayo hasta algo más que la mitad. Uno(a) de ustedes tapa con su dedo pulgar la boca del tubo de ensayo con agua y lo invierte, metiéndolo en seguida a la botella. Por último, tapen la botella de manera hermética. El tubo queda, entonces, invertido dentro de la botella.

Observen con atención el sistema y noten que el tubo de ensayo sumergido en el agua se comporta como un submarino. Observen con atención el aire que está dentro del tubo, mientras uno(a) de ustedes aprieta con sus manos la botella. Dejen de apretar, suavemente, y observen el comportamiento del tubo de ensayo. A continuación, respondan:

b) ¿Qué ocurre con el tubo de ensayo cuando se aprieta la botella? ¿Qué ocurre cuando se deja de apretar la botella? ¿Por qué?

c) ¿Cómo actúa sobre el fluido la fuerza que se aplica en las paredes de la botella?

d) Sitúen el tubo al fondo de la botella. ¿Qué fuerzas actúan sobre el tubo en este caso?

e) Cuando sitúan el tubo en el centro de la botella, ¿qué fuerzas actúan sobre él?

f) ¿Es constante el peso del objeto? ¿Por qué? ¿Cómo se relaciona su peso con la fuerza que el agua ejerce sobre él? A partir de su respuesta, evalúen la validez de su hipótesis.

Para finalizar la actividad, preparen un informe sobre su trabajo según las indicaciones de su profesor(a) y luego presenten a sus compañeros(as) cuáles fueron sus hallazgos.

Sección 6: Principio de Arquímedes

Page 35: FLUIDOS - FISICA

Física 3° Año Medio146

Fluidos

Evaluación intermediaPARTE I: Problema de planteamiento

1 En esta sección aprendimos que todos los cuerpos contenidos en un fluido experimentan una fuerza lla-mada empuje.

a) ¿Cómo se relaciona esta fuerza con las propiedades del cuerpo y del fluido? Explica.

PARTE II: Análisis

2 Los grandes barcos trasatlánticos pesan mucho, ya que están hechos con materiales muy densos comparados con el agua, como el hierro o el acero. Entonces ¿cómo se explica que puedan flotar?

Indagación N°17

¿Cómo sería el tamaño de dos pompas de jabón de distinto diámetro si se unen a través de un tubo?

Para responder la pregunta planteada en el título de esta actividad, se propone la siguiente hipótesis:

Las pompas esféricas llegarán a ser del mismo tamaño, ya que la mayor pre-sión de la pompa grande se equilibrará con la menor presión de la pequeña, quedando finalmente con igual diámetro.

¿Cómo podemos poner a prueba esta hipótesis?

a) Junto a un compañero o una compañera, diseñen un procedimiento experi-mental que les permita, a través de un modelo, poner a prueba la hipótesis para evaluar si es una explicación aceptable o debe ser descartada. Dibujen su montaje experimental y describan brevemente, pero con precisión, el procedimiento que sugieren.

Procuren que el procedimiento experimental propuesto sea factible de reali-zar en una hora de clases; es decir, que incluya el uso de materiales de fácil adquisición o construcción y tiempos razonables para la observación y el análisis de sus resultados.

b) Para finalizar, elaboren un informe de dos páginas según las indicaciones que les dé su profesor(a).

Page 36: FLUIDOS - FISICA

147

Capítulo 3: Hidrostática

Tensión superficial y capilaridad

¿Qué tienen en común las burbujas de jabón con las gotas de rocío en una tela de araña?

Figura 6.11. Este insecto es un “zapa-tero de agua” o Gerris lacustris. ¿Cómo logra mantenerse sobre la superficie del agua sin hundirse?

Figura 6.10. Burbujas de jabón y gotas de rocío en una tela de araña, ¿qué tienen en común?

Las gotas de rocío y las burbujas de jabón son líquidos que adoptan una forma particular. ¿Por qué en ambos casos el líquido adopta una forma esférica? ¿En qué otros casos observamos estas características?

A continuación, estudiaremos la propiedad que explica estos fenómenos y qué hace que la superficie de un líquido tienda a comportarse como si fuera una delgada película elástica. Se trata de la tensión superficial. Gracias a ella, algunos insectos pueden desplazarse por la superficie del agua sin hundirse.

En combinación con las fuerzas que se dan entre los líquidos y las superficies sólidas, la tensión superficial produce otro fenómeno muy importante: la capilaridad, que, entre otras cosas, es esencial para el crecimiento de las plantas.

TENSIÓN SUPERFICIALLa tensión superficial es la propiedad que hace que la superficie de los líquidos tienda a contraerse, comportándose como si fuera una membrana elástica.

De esta manera se explica la forma esférica de las gotas de los líquidos.

Figura 6.12. ¿Cómo se puede formar esta superficie elástica y tan delgada a partir de una solución jabonosa?

Sección 6: Principio de Arquímedes

Page 37: FLUIDOS - FISICA

Física 3° Año Medio148

Fluidos

Las gotas de agua o de aceite son esféricas porque sus superficies tienden a contraerse y a hacer que cada gota adopte una forma de superficie mínima. Esa forma es la esfera, que es el cuerpo geométrico que ocupa la menor superficie para un determinado volumen.

Técnicamente, la tensión superficial es una fuerza por unidad de longitud o, equivalentemente, el trabajo por unidad de área necesario para aumentar la superficie del líquido. En otras pala-bras, la forma esférica de las gotas de los líquidos es la forma que minimiza su energía.

Esta fuerza tiene una explicación a nivel microscópico molecular. Como se muestra en la Figura 6.13, las moléculas en el interior de un líquido experimentan la fuerza de atracción de las otras moléculas. Esta fuerza es de origen eletromagnético y se conoce como fuerza de cohesión.

Para cada molécula bajo la superficie del líquido, las fuerzas de atracción actúan en todas direcciones y, como resultado, no hay una fuerza neta sobre cada molécula, es decir, las fuerzas se en-cuentran equilibradas entre sí.

En cambio, sobre las moléculas en la superficie del líquido, las fuerzas laterales están equilibradas, pero las fuerzas verticales están desequilibradas, ya que no hay otras moléculas de líquido por encima de ellas.

Es la acción de este desequilibrio de fuerzas sobre las moléculas superficiales la que provoca la tensión superficial de un líquido. En conjunto, las moléculas superficiales se comportan como una pequeña película o pantalla de protección del líquido, impidiendo que, dentro de ciertos límites, este se rompa por tirones externos o compresiones.

Consideremos la situación que se muestra en la Figura 6.14. Si sobre la superficie del agua se deposita una aguja o un clip de acero, secos, quedan suspendidos en la superficie del líquido. ¿Cómo es posible si la densidad del acero es casi ocho veces la densidad del agua?

Son las fuerzas moleculares a nivel microscópico las que equili-bran el pequeño peso de la aguja acostada en el agua o del clip, actuando desde un punto de vista macroscópico como una super-ficie elástica.

¿Se puede cambiar la tensión superficial de un líquido?

Figura 6.13. Una molécula bajo la superficie es atraída por igual en todas direcciones por las otras moléculas que la rodean. En cambio, una molécula en la superficie solo es atraída hacia los lados y hacia abajo. Esto proporciona la tensión superficial suficiente para soportar el peso del mosquito.

Figura 6.14. ¿Qué forma adopta el agua en los contornos del objeto? ¿El agua moja o no moja al clip? ¿Cómo se relaciona este ejemplo con el uso de detergentes para el lavado de ropa?

Page 38: FLUIDOS - FISICA

149

Capítulo 3: Hidrostática

Tabla 6.1. Tensión superficial de al-gunos líquidos. En general, la tensión superficial depende de la temperatura del líquido. Todos los valores están medidos en relación a la superficie entre el líquido y el aire.

Para medir la tensión superficial, podemos usar un procedimiento sencillo, conocido como método de Du Noüy, por el bioquímico y matemático francés que lo inventó. Consiste en aplicar una fuerza hacia arriba sobre una anillo de alambre amarrado con un hilo, el cual se levanta suavemente desde la superficie del líquido. En estas condiciones, la tensión superficial impide que el anillo se levante inmediatamente.

Como la tensión superficial (γ) se define como la fuerza por unidad de longitud, podemos obtenerla midiendo la longitud del anillo de alambre (L) y la fuerza (F) aplicada para separarlo del agua, lo cual requiere un instrumento de precisión. De este modo:

γ =FL2

(6.8)

Según esto, en el S.I. la tensión superficial se expresa en Nm

.

Líquido Temp. (°C)

γNm

Acetona 20 23,7 · 10-3

Eter etílico 20 17 · 10-3

Etanol 20 22,27 · 10-3

Glicerol 20 63 · 10-3

Mercurio 15 487 · 10-3

Agua 0 75,64 · 10-3

Agua 25 71,97 · 10-3

Agua 50 67,91 · 10-3

Agua 100 58,85 · 10-3

Figura 6.15. Tensiómetro de Du Noüy. Una argolla de alambre se levanta en una solución para medir su tensión superficial.

¿Por qué se require un instrumento de precisión para medir la fuerza aplicada por el anillo de Du Noüy?

Ejemplo 7

Un anillo de 10 cm de diámetro ejerce una fuerza de 0,045 N hacia arriba sobre la superficie del agua en un recipiente.

a) ¿Cuál es la tensión superficial del fluido?

Sección 6: Principio de Arquímedes

Page 39: FLUIDOS - FISICA

Física 3° Año Medio150

Fluidos

a: Como la superficie ejerce una tensión sobre el radio in-terno y el radio externo del anillo, la fuerza por unidad de longitud se expresa, de acuerdo a la ecuación (6.8), del siguiente modo:

γ

γπ

γ

γ

=

=⋅ ⋅ ⋅( )

=⋅ ⋅

=

FL

FR

Nm

2

2 2

0 0454 3 14 0 05

0 07

,, ,

, 11Nm

Es decir, la superficie ejerce una tensión de 0,071 N por cada metro de longitud.

CAPILARIDADCuando se sumerge en agua el extremo de un tubo de vidrio, cuyo diámetro interno es pequeño, el agua es capaz de ascender por él espontáneamente. En un tubo de 0,5 mm de diámetro, por ejemplo, el agua asciende alrededor de 5 cm por el interior del tubo. Este ascenso del agua por un tubo fino y hueco se conoce como capi-laridad, ya que a ese tipo de elemento se le llama capilar (palabra que deriva del latín y significa cabello).

La Figura 6.16 explica por qué en la ecuación (6.8) usamos el denominador 2L. La razón es que el líquido se adhiere al alambre por su cara interna y por su cara externa, de modo que la longitud en contacto con la superficie es aproxi-madamente el doble del perímetro del anillo.

Un análisis más riguroso de la Figura 6.16, en términos del trabajo por uni-dad de area realizado al elevar el anillo una distancia ∆h , conduce al mismo resultado:

γπ

=⋅ ⋅F

R4 (6.9)

Donde R es el radio medio del anillo.

Figura 6.17. Como todos los tubos están abiertos en el extremo superior, por el principio de Pascal el nivel del líquido en todos ellos debería ser el mismo. Sin embargo, se observa el efecto de la capilaridad, el cual es mayor cuando el diámetro del tubo es menor.

Figura 6.16. Anillo de Du Noüy. Se puede observar que hay dos superficies del líquido adheridas al alambre, una superficie cilíndrica interna (Si) y una superficie cilíndrica externa (Se). Ri y Re son los radios interno y externo del anillo, respectivamente. r es el radio del alambre.

Vista superior del anillo

Superficie del líquido

Corte frontal del anillo

Re

Ri

R

Si SeΔh

2r

Page 40: FLUIDOS - FISICA

151

Capítulo 3: Hidrostática

La capilaridad ocurre porque, como vimos en el apartado anterior, en cierto modo las moléculas del líquido son “pegajosas”.

La atracción entre moléculas de la misma sustancia es llamada cohesión. La atracción entre moléculas de sustancias diferentes se conoce como adhesión.

Observemos la secuencia de la Figura 6.18:

Figura 6.18. Un tubo capilar de vidrio se introduce en un líquido. En la secuencia, (b) se observa como el líquido inicialmente se adhiere al vidrio por la superficie interna y externa del capilar, (c) luego, la tensión superficial hace que la película adherida se contraiga, redondeando sus contornos y (d) la película de la superficie interior se contrae más, elevando el líquido hasta que su peso queda equilibrado con la fuerza de adhesión.

Al introducir el capilar de vidrio en el líquido, la fuerza de adhesión hace que el fluido suba por las paredes del tubo, mientras la tensión superficial tiende a contraer la película de líquido, redondeando los contornos dentro y fuera del capilar. La superficie del líquido en el interior se contrae más y esto eleva al líquido por el tubo, hasta que su peso es equilibrado por la fuerza de adhesión. Así, el agua que asciende por un tubo más delgado tiene un peso menor, por lo que alcanza más altura.

La relación entre la fuerza de cohesión de un líquido y la fuerza de adhesión que presenta ante un sólido, determina si el líquido se esparce o no por la superficie del sólido; es decir, si lo moja o no.

Si la fuerza de cohesión de una gota de líquido es menor que la fuerza de adhesión entre sus moléculas y las de la superficie del sólido, entonces la gota se esparce por el sólido, mojándolo.

Figura 6.19. Es algo sabido que las plantas consiguen el agua y los nutrientes del suelo por medio de las raíces, luego transportan este material (savia bruta) a través del tallo hasta las hojas, donde realizan la fotosíntesis gracias a la clorofila y la luz solar, y que por último distribuyen la glucosa, azúcares y aminoácidos obtenidos (sa-via elaborada) por toda su estructura. Pero, ¿cómo hacen para transportar el agua con las sustancias disueltas? ¿Se contraen? ¿Hay alguna especie de mecanismo de bombeo?

(a) (b) (c) (d)

Sección 6: Principio de Arquímedes

La ley de Jurin define la altura máxima que alcanza una columna de fluido que asciende por capilaridad. La altura h de la columna, en metros, está dada por la ecuación:

h g r= ⋅ ⋅⋅ ⋅

2 γ θρ

cos (6.10)

Donde γ es la tensión superficial, θ es el ángulo de contacto, ρ es la densidad del líquido, g es la aceleración de gravedad y r es el radio del tubo capilar.

Page 41: FLUIDOS - FISICA

Física 3° Año Medio152

Fluidos

Por el contrario, si la fuerza de cohesión es mayor, el sólido no se moja.

Una gota de mercurio líquido, por ejemplo, no se esparce en la superficie limpia de un vidrio. Lo mismo ocurre con una gota de lluvia que cae sobre un automóvil recién encerado: la gota no moja al auto, más bien resbala debido a la nula adherencia.

La capacidad de adhesión de una gota con una superficie sólida versus la capacidad de cohesión de las moléculas del líquido se puede cuantificar a través de un ángulo fácilmente medible denominado ángulo de contacto. En la Figura 6.20, se muestran las diferentes formas de contacto de una gota de líquido con un sólido.

Figura 6.21. El mercurio líquido no moja la superficie de contacto porque la fuerza de cohesión entre sus moléculas es mayor que la fuerza de adhesión con la madera.

H2O Hg

Figura 6.22. ¿Cómo se puede expli-car la diferencia de curvatura entre el agua y el mercurio al interior de un capilar?

θ θ θ

aire aire aire

sólidosólidosólido

Figura 6.20. Ángulo de contacto de tres líquidos diferentes sobre la superficie de un sólido.

En la Figura 6.20, se puede observar que si el ángulo de contacto es θ < 90º , entonces la gota se esparce y moja al sólido. Si el ángulo de contacto es 90 180º º≤ <θ , el líquido no se esparce y, por lo tanto, no moja al sólido. Cuando la gota de agua moja la superficie sólida, se debe a que la fuerza de adhesión es más grande que la fuerza de cohesión, y viceversa.

De acuerdo a lo anterior, un líquido podría no mojar el interior de un capilar. En el caso en que el líquido logra mojar las pare-des del tubo, porque la fuerza de adhesión es mayor que la de cohesión, se produce una concavidad hacia arriba en el fluido o menisco cóncavo. Como se muestra en la Figura 6.22 para el caso del agua, el líquido asciende por el tubo, en el efecto que llamamos capilaridad.

Si el líquido no moja a las paredes del tubo, se produce una con-cavidad hacia abajo o menisco convexo. En este caso, no hay capilaridad, sino, al contrario, el líquido desciende por el tubo, como se muestra en la Figura 6.22, para el caso del mercurio.

Es importante señalar que el mercurio que encontramos en termómetros, ampolletas de bajo consumo y otros, es altamente tóxico por contacto, inha-lación o ingestión. Cuando se rompe alguno ellos, es indispensable tomar precauciones:Ventilar la zona al menos por 15 minutos. Utilizar guantes desechables y poner los trozos grandes en bolsas plásticas. Con toallas de papel húmedas recoger los residuos más pequeños. No utilizar aspiradora ni escoba, ya que esparcirá material peligroso en el aire. No eliminar por el desagüe. Poner los elementos utilizados en una segunda bolsa gruesa, sellarla y rotularla como “Sustancia peligrosa: contiene mercurio y vidrio”. Idealmente llevarlo a un depósito de sustancias peligrosas o eliminarlo en un basurero de forma segura. Lavar cuidadosamente las manos.

Page 42: FLUIDOS - FISICA

153

Capítulo 3: Hidrostática

LA CAPILARIDAD OCURRE EN TODAS PARTESLa capilaridad es un fenómeno fundamental en muchas situaciones naturales y artificiales.

Antes de alcanzar las raíces de las plantas, gracias a la capilaridad el agua que cae sobre la tierra se distribuye por los microespa-cios de aire que quedan entre las partículas del suelo. Después, el transporte de agua y otras sustancias desde las raíces hasta las hojas en las plantas es un problema de la fisiología vegetal en el cual la capilaridad juega un rol crucial.

El agua que se introduce por las raíces, a través de los pelos ra-diculares, penetra en un sistema de células interconectadas que forman el tejido de la planta y que se extienden desde las mismas raíces hasta las hojas, a través del tronco o tallo. Este tejido leño-so, llamado xilema, está formado por varios tipos de células. El ascenso de savia bruta se ve favorecido por el reducido tamaño de los vasos leñosos a los que se adhieren las moléculas de agua, pues el ascenso es más eficaz cuanto menor es el diámetro del vaso, es decir, por capilaridad.

Sin embargo, la capilaridad no es suficiente para elevar el agua hasta todos los lugares de la planta. Varios procesos adicionales se requieren para que esto suceda, entre los cuales el más importante es la evaporación de las moléculas del agua a través de las hojas. Como las moléculas de agua tienden a unirse unas con otras gracias a su fuerza de cohesión, cuando una molécula se evapora a través del poro de una hoja, se ejerce un pequeño empuje a las moléculas adyacentes, lo que reduce la presión en las células leñosas y atrae agua de las células contiguas. Este efecto de llamada se extiende por todo el trayecto hasta las raíces y se suma al efecto de la capilaridad.

En el sistema circulatorio de nuestros cuerpos también ocurre el fenómeno capilar. Unos diez mil millones de capilares se entrela-zan por todos los tejidos del cuerpo, suministrando sangre a todas las células. Son los vasos sanguíneos más pequeños, de tamaño microscópico, y contienen menos del cinco por ciento del volumen total de la sangre que circula.

En objetos tecnológicos encontramos capilaridad en muchos casos: esponjas, toallas de papel, telas, mecheros de alcohol, plumones de tinta, bolígrafos, etc. Incluso los muros de una construcción se humedecen y deterioran porque el agua asciende por su interior debido al mismo fenómeno.

Figura 6.23. Tejido xilemático. Estos orificios poseen en su interior una membrana conformada por una red de micro fibras elásticas que actúan como una válvula capilar.

Figura 6.24. La mecha funciona como un elemento absorbente del alcohol, debido al efecto de capilaridad.

Figura 6.25. En un muro, la capila-ridad provoca que el agua ascienda internamente.

Sección 6: Principio de Arquímedes

SUBSUELO HÚMEDO

Page 43: FLUIDOS - FISICA

Física 3° Año Medio154

Fluidos

A partir de la lista de conceptos relevantes (CR) y frases conectoras (FC), completa en tu cuaderno el mapa conceptual de la figura.

Conceptos Relevantes (CR) Frases Conectoras (FC)A Adhesión I Producen laB Volumen II Es el cuociente entreC Aceleración de gravedad III Cumplen elD Densidad de fluido IV Explica elE CohesiónF Empuje

Síntesis

1

Fluidos

DesafíoCuando hayas terminado esta

actividad, vuelve a leer el texto de la sección, con mucha aten-ción, y genera tu propio mapa

conceptual.

2

Tienen

Tensión superficial

Masa

7

Principio de Arquímedes

Flotación de los cuerpos

Capilaridad

Genera la

9

8

5

Produce

Volumen del fluido

desplazado

3

10

6

4

Es el producto de

Page 44: FLUIDOS - FISICA

155

Capítulo 3: Hidrostática

Preguntas y ejercicios propuestos

1 ¿En qué situaciones de tu vida diaria puedes apreciar el principio de Arquímides?

2 ¿Por qué si se ponen agua y aceite en un recipiente, el aceite permanece arriba del agua?

3 ¿Cómo se puede cambiar la tensión superficial de un líquido?

4 ¿En qué casos un líquido no moja la superficie? Explica.

5 ¿Cómo se relaciona el tejido xilemático de las plantas con la capilaridad?

6 Un objeto sólido de 30 cm3 se sumerge comple-tamente en un fluido. (a) ¿Cuál es el módulo de la fuerza de empuje que el objeto experimenta si el fluido es agua? (b) ¿Cuál es el empuje si el fluido es mercurio?

7 Cómo muestra la Figura 6.26, un objeto cúbico flota en equilibrio sobre la superficie del agua con el 60% de su volumen sumergido. (a) ¿Cuál es el módulo de la fuerza resultante que actúa sobre el objeto? (b) ¿Cuál es su densidad?

Figura 6.26

8 Un trozo de metal tiene una masa de 180 kg y un peso aparente de 1 400 N, cuando se le sumerge completamente en agua. (a) ¿Cuánto pesa en el aire el trozo de metal? (b) ¿Cuál es la densidad del metal?

9 Un cuerpo cuyo volumen es de 900 cm3 tiene un peso aparente de 3,6 N cuando se le sumerge en alcohol ( ρalcohol

kgm

= ⋅0 8 1033, ). (a) ¿cuál es su

peso en el aire y su densidad?

10 Una medalla tiene una masa de 0,1 kg y su peso aparente al sumergirla completamente en agua es de 0,94 N. (a) ¿Cuál es el módulo del peso de la medalla? (b) ¿Cuál es su volumen? (c) ¿Es de oro la medalla? ( ρoro

kgm

= ⋅19 3 1033, )

11 De acuerdo a la Figura 6.27, sobre una balanza hay un recipiente lleno de agua, cuya masa total es de 1 kg. Un cubo de aluminio de 1 cm de arista se suspende en un dinamómetro de resorte y se sumerge en el agua, de manera que la mitad del bloque queda afuera del líquido. (a) ¿Qué me-dida registra el dinamómetro cuando el cubo de aluminio está en el agua? (b) ¿Qué masa mide la balanza en esta misma situación?

ρAlkgm

= ⋅2,698 1033

Figura 6.27

12 Un bloque de madera de pino ( ρ = 300 3kgm

) tiene las siguientes dimensiones: 10 cm, 40 cm y 5 cm. El bloque flota en una piscina con agua, con su cara de mayor área paralela a la superficie del líquido. (a) ¿Cuál es el espesor del bloque que sobresale del agua? (b) ¿Qué masa extra mínima es necesario agregar al bloque para que quede completamente sumergido?

Sección 6: Principio de Arquímedes

Page 45: FLUIDOS - FISICA

Física 3° Año Medio156

Fluidos

13 Observa la Figura 6.28. Un bloque de madera ( ρmadera

kgm

= 600 3 ) flota en agua y tiene las si-guientes dimensiones: 20 cm, 20 cm y 10 cm. (a) ¿Qué espesor debe tener una lámina de hierro de 20 cm por 20 cm que, superpuesta a la madera, permite que la parte superior del bloque quede nivelada con la superficie del agua? (b) ¿Qué es-pesor debe tener la lámina de hierro si se adhiere al cubo de madera por la parte inferior, de manera que se cumpla la misma condición anterior? ( ρhierro

kgm

= 7860 3 )

madera madera

aguaagua

hierrohierro

Figura 6.28

14 En la corte de un rey, el año 1312, se duda de que una de sus coronas sea de oro, y se afirma que es posible que esté hecha de plomo y recubierta con oro. Cuando se mide la masa, manera normal en aire, la balanza registra 0,475 kg. Cuando se sumerge completamente en agua la masa medida es de 0,437 kg. (a) ¿Cuál es la densidad de la corona? (b) ¿Qué porcentaje de su volu-men es efectivamente de oro? ( ρoro = 19300 3

Kgm

, ρplomo = 11300 3

Kgm )

15 Un anillo de 4 cm de radio, hecho de alambre delgado, se encuentra horizontalmente sumergido en agua a 25° C. (a) ¿Cuál es la fuerza mínima necesaria para sacarlo y con ello superar la tensión superficial?

16 Un tensiómetro de Du Noüy de 5 cm de diámetro se usa para determinar la tensión superficial de un fluido líquido. Si se mide una fuerza de 2,3 · 10-2 N, ¿cuál es la tensión superficial del fluido?

17 Un tubo capilar de diámetro interno 0,1 mm contiene agua a una temperatura de 25° C. Si el ángulo de contacto entre el agua y el vidrio es θ = 0°, ¿hasta qué altura asciende el agua por el interior del capilar?

18 De acuerdo a la Figura 6.29, se vierte agua a 25°C dentro de un tubo vertical en forma de U donde los brazos tienen diámetros internos diferentes. Si el diámetro de uno de ellos es igual a 0,6 mm y el diámetro del otro es de 1,2 mm, ¿cuál es la diferencia de altura que alcanza el agua entre los dos brazos del tubo? (Considere que el ángulo de contacto es θ = 0°)

Figura 6.29

19 Considerando que la tensión superficial se puede definir como la cantidad de trabajo por unidad de área, necesario para aumentar la superficie de un líquido, ¿en qué factor se incrementa la energía de la superficie de una burbuja de jabón perfectamente esférica, que aumenta su diámetro de 2 cm a 6 cm?

20 Un prisma triangular de hielo flota en agua de mar con la cúspide sumergida. Demuestra que el volumen del hielo satisface la siguiente rela-ción:

VVsumergido

total

hielo

aguademar=

ρρ

Page 46: FLUIDOS - FISICA

157

Capítulo 3: Hidrostática

Evaluación final de la sección

PARTE I: Anota en el recuadro el número del concepto que corresponde a su descripción o definición.

Magnitud Descripción / definición1 Empuje Densidad del cuerpo es menor que la del fluido.2 Flotación Fuerza igual al peso del fluido desplazado.3 Hundimiento Ascenso del fluido debido a la tensión superficial.4 Capilaridad Densidad del cuerpo es mayor que la del fluido.

PARTE II: Indica si el enunciado es verdadero o falso. Expresa en tu cuaderno la justificación de tus respuestas.

V o F1 Un objeto sumergido totalmente en el agua tiene la misma masa que en el aire.2 Un objeto sumergido totalmente en el agua tiene el mismo peso aparente que en el aire.3 La fuerza de empuje es inversamente proporcional a la masa del fluido desalojado.4 En la Capilaridad, la adherencia del fluido al tubo es menor que la cohesión entre sus moléculas.5 El ángulo de contacto mide la relación entre la adherencia y la cohesión.

PARTE III: Responde las siguientes preguntas, marcando la alternativa correcta.1 Si al sumergir completamente un cuerpo en

agua (densidad = 1000 kg/m3), experimenta un empuje de 100 N, ¿cuál es el volumen del cuerpo? a) 0,1 m3 b) 0,01 m3 c) 0,01 cm3 d) 0,02 cm3

2 Una esfera de aluminio de densidad 2,7 g/cm3 y de 4 cm de radio, se coloca en agua de mar cuya densidad es de 1025 kg/m3. ¿Qué le ocurre al objeto?a) Flota en la superficie. b) Flota en el interior del fluido. c) Se hunde hasta el fondo. d) Nada de su cuerpo se sumerge.

3 Un cubo de hielo de 1cm de lado está flotando en agua (densidad = 1 000 kg/m3). Si la densidad del hielo es de 840 kg/m3, ¿qué porcentaje de su volumen está sumergido?a) 84 % b) 0,16 % c) 0,84 % d) 16 %

4 Un contenedor posee hielo solo en su parte inferior, y agua en su parte superior. Si antes de calentarlo y derretir el hielo (sin hervir), el contenedor marcaba 500 ml. ¿Cuánto marcará después de que los hielos se derritan?a) 500 ml.b) Menos de 500 ml.c) Más de 500 ml.d) Depende de la temperatura del agua.

Sección 6: Principio de Arquímedes

Page 47: FLUIDOS - FISICA

Física 3° Año Medio158

Fluidos

Antes de empezar...

1 Un flujo estacionario, ¿se mue-ve?

2 Si el diámetro de una tubería se reduce de manera uniforme, ¿la cantidad de agua que sale por el extremo delgado es igual a la cantidad de agua que entra por el otro extremo?

3 ¿Cuál es la característica básica de un flujo estacionario?

4 ¿Qué le ocurre a la rapidez del flujo de agua cuando pasa por la salida estrecha de una manguera?

5 ¿De qué principio fundamental se deriva la ecuación de Ber-noulli?

6 ¿En qué sectores de un río el agua fluye más rápido?

7 ¿Cómo se explica que una pelota que viaja rotando pueda curvarse hacia un lado en su trayectoria?

8 ¿Para qué sirve el medidor de Venturi?

9 ¿Todo objeto que cae aumenta su rapidez?

10 ¿Cómo vuela un aeroplano? ¿Qué tiene en común con el diseño de los seres voladores?

“Un pájaro es una máquina que funciona según las leyes de la matemática. Está al alcance del hombre reproducir esa máquina con todos sus movimientos, aunque no con su misma fuerza... A esa máquina construida por el hombre solo le faltaría el espíritu del pájaro, y ése es el que el hombre ha de imitar con su propio espíritu”.

Leonardo da Vinci (1452-1519), humanista e inventor italiano.

Capítulo 4

Page 48: FLUIDOS - FISICA

159

Capítulo 4: Hidrodinámica

Indagación N°18¿Por qué se angosta un flujo de agua en la medida que cae?

PARTE I: Trabajo personal

Cuando se riega un jardín con una manguera manual y se necesita que el chorro de agua tenga mayor alcance horizontal, la experiencia sugiere que se apriete o estrangule la boca de la manguera, por donde sale el agua. Efectivamente, se logra así un mayor alcance (imagen 7.1).

Sin embargo, cuando se produce un flujo regular en la caída de agua de una llave, las cosas no son tan intuitivas.

a) En una caída vertical de agua desde una llave (imagen 7.2), el flujo se vuelve cada vez más delgado en la medida que baja. Si supone-mos que el líquido se compone de muchas “partículas”, ¿qué tipo de movimiento experimentan estas partículas?

b) ¿Por qué se angosta el flujo de agua en la medida que cae?

PARTE II: Trabajo en equipo

Junto a un compañero o una compañera, contrasten sus respuestas y argumenten a favor o en contra de ellas.

A continuación, elaboren una hipótesis en conjunto que dé respuesta a la segunda pregunta.

a) Registren la hipótesis en sus cuadernos e identifiquen cuáles son las variables observables que pueden medir y/o controlar.

b) Una vez planteada su hipotesis, diseñen un procedimiento experimental que les permita ponerla a prueba, para evaluar si es una explicación aceptable o debe ser descartada. Dibujen su montaje experimental y describan brevemente, pero con precisión, el procedimiento que sugieren.

Procuren que el procedimiento experimental propuesto sea factible de realizar en una hora de clases; es decir, que incluya el uso de materiales de fácil adquisición o construcción y tiempos razonables para la obser-vación y el análisis de sus resultados.

c) Para finalizar, elaboren un informe de dos páginas según las indica-ciones que les dé su profesor(a).

Imagen 7.1. Estrangulamos la man-guera para tener mayor alcance con el chorro de agua.

Imagen 7.2.

Recuerden que una hipótesis es una explicación posible que se supone cierta hasta que pueda ser contrastada empíricamente. Por esta razón, es fundamental que la hipótesis se refiera a un número reducido de variables observables y de algún modo medibles, que eventualmente pueden ser controladas en un experimento.

Page 49: FLUIDOS - FISICA

Física 3° Año Medio160

Fluidos

Indagación N°19¿Por qué un vehículo en movimiento “atrae” a los objetos alrededor?

PARTE I: Trabajo personal

Es típico usar la presión de un fluido para “empujar” algún objeto. Por ejemplo, cuando soplamos un objeto liviano para moverlo. Sin embargo, cuando pasa junto a nosotros un vehículo moviéndose rápidamente, en vez de empujarnos por el movimiento del aire que provoca, parece atraernos hacie él. En estas condiciones, ¿por qué el movimiento del vehículo nos “atrae”, en vez de empujarnos?

a) Plantea una hipótesis que dé respuesta a esta pregunta y regístrala en tu cuader-no.

PARTE II: Observación compartida

Reúnete con un compañero o compañera para compartir sus hipótesis. Comenten y argumenten a favor o en contra de ellas. Luego, sigan con atención la demos-tración que dirigirá su profesor(a) y respondan en su cuaderno las siguientes preguntas.

a) ¿Qué les ocurre a los globos cuando se sopla frontalmente a uno de ellos?

b) ¿Cómo se mueven los globos cuando se sopla entre ellos?

c) ¿A qué se debe la diferencia entre ambos casos?

PARTE III. Trabajo en equipo

En esta parte de la actividad, junto a tu compañero(a) realizarán un sencillo ex-perimento, para el cual necesitan: una hoja de cuaderno, un embudo y una pelota de ping-pong.

Uno(a) de ustedes coloca el embudo como muestra el imagen 7.3a y sopla fuer-temente, al mismo tiempo que deja de sostener la pelota de ping pong. A conti-nuación, toman una hoja de papel, como en el imagen 7.3b, y soplan fuertemente sobre ella.

a) ¿Por qué la pelota no cae y queda suspendida? ¿Por qué se levanta la hoja al soplar sobre ella?

b) ¿Cómo se relacionan estas observaciones con el problema inicial? ¿Qué tienen en común las dos situaciones?

c) Comparen sus respuestas con la hipótesis inicial que cada uno planteó y es-criban sus conclusiones.

(a)

(b)

Imagen 7.3

Page 50: FLUIDOS - FISICA

161

Capítulo 4: Hidrodinámica

Fluidos en movimiento

Hasta aquí, nuestro estudio de los fluidos se ha limitado a los fluidos estáticos. Ahora nos concentraremos en el estudio de los fluidos cuando están en movimiento, y para ello haremos uso de algunos de los conceptos que aprendiste en las secciones anteriores, como densidad y presión. En otras palabras, describiremos la dinámica de los fluidos en función de sus propiedades globales.

Sin embargo, cuando se trata de fluidos reales, no es fácil describir su movimiento, ya que se producen fenómenos muy complejos que todavía no se comprenden por completo. Por ejemplo, ¿has observado el flujo de un canal de agua de regadío o el movimiento de las partículas de humo en el aire? En ocasiones aparecen com-portamientos impredecibles, muy difíciles de explicar.

Por esto, como es habitual en física, haremos uso de un mo-delo simplificado que, a pesar de sus limitaciones, resulta muy efectivo para entender el comportamiento de los fluidos en movimiento.

FlujoConsideremos el movimiento de un fluido de un modo idealizado. De acuerdo a esto, el flujo de un fluido puede ser de dos tipos.

Por una parte, se dice que un flujo es estacionario o laminar, cuando cada partícula del fluido sigue una trayectoria uniforme que no se cruza con la trayectoria de las otras partículas. De esta manera, las partículas forman capas o láminas y se mueven sin que haya mezcla significativa de partículas de fluido vecinas.

Por otra parte, cuando el fluido se mueve con una rapidez superior a cierta rapidez crítica, el flujo se vuelve turbulento. Este tipo de flujo se caracteriza por ser irregular debido a la presencia de remolinos, como ocurre en las zonas en que los ríos se encuentran con obstáculos.

Para caracterizar la fricción interna de un fluido cualquiera se usa un parámetro conocido como viscosidad. Cuando un fluido es más viscoso, entonces hay mayor fricción entre sus capas, lo que dificulta su movimiento, de manera análoga a la acción de la fuerza de roce por deslizamiento entre dos superficies.

Figura 7.1. En un túnel de viento, se puede observar el flujo estacionario del aire alrededor del vehículo. Las líneas corresponden a las llamadas líneas de flujo, que en este caso se hacen visibles con partículas de humo.

7

Figura 7.2. Flujo turbulento de vapor. En la imagen se puede apreciar la formación de remolinos o vórtices.

Sección 7: Fluidos en movimiento

Sección

Page 51: FLUIDOS - FISICA

Física 3° Año Medio162

Fluidos

Así, de acuerdo a nuestra motivación inicial por hacer uso de un modelo simplificado para estudiar la dinámica de los fluidos, consideraremos las siguientes propiedades de un fluido ideal:

• Fluido no viscoso. Es decir, despreciaremos los efectos de la viscosidad. Según esta suposición, las láminas constituyentes del fluido no interactúan entre sí, y tampoco interactúan con las paredes del conducto en el que fluyen.

• Fluido incompresible. En general, los fluidos pueden ser com-presibles. El aire encerrado en una jeringa, por ejemplo, es un gas evidentemente compresible. Sin embargo, en esta sección solo consideramos fluidos homogéneos incompresibles, cuya densidad es constante, independientemente de la presión. Este es el caso de cualquier líquido a temperatura constante que se mueve en un conducto, y también el de algunos gases.

• Flujo estacionario. Es decir, cada partícula del fluido sigue una trayectoria uniforme que no se cruza con la trayectoria de las otras partículas.

• Flujo irrotacional. Es decir, en el fluido no se producen re-molinos o vórtices.

Líneas de flujo y ecuación de continuidadLa trayectoria que sigue una “partícula” de fluido en condiciones de flujo estacionario se llama línea de flujo. Un flujo estacionario se caracteriza porque todas las líneas de flujo se presentan bien definidas y separadas unas de otras, de manera que nunca se cruzan entre sí.

En otras palabras, las líneas de flujo son líneas imaginarias que representan las huellas de las partículas del fluido.

En estas condiciones, la dirección del vector velocidad de cada par-tícula, en un punto determinado del fluido, es un vector tangente a la línea de flujo que pasa por ese punto, como muestra la figura 7.3.

Veremos a continuación que el distanciamiento de las líneas de flujo está relacionado con la velocidad del fluido, de manera que cuando las líneas de flujo se acercan entre sí, la velocidad de las partículas del fluido es mayor que cuando las líneas de flujo están más separadas.

Observa la Figura 7.4. ¿Por qué debería aumentar la velocidad del fluido en las partes más estrechas del conducto?

Figura 7.3. Las líneas representan las líneas de flujo de un fluido en ré-gimen estacionario. Una partícula del fluido situada en P sigue una de estas líneas, de manera que su velocidad es tangente a la línea de flujo en toda su trayectoria.

Figura 7.4. Ejemplos de flujo estaciona-rio a través de conductos de diferentes formas, en los que las líneas de flujo se aproximan o se alejan entre sí, indican-do que la rapidez del fluido aumenta o disminuye, respectivamente.

P

Page 52: FLUIDOS - FISICA

163

Capítulo 4: Hidrodinámica

Consideremos un fluido ideal que se mueve en el interior de una tubería cuya sección transversal no es uniforme. Si en la tubería no hay agujeros, no es posible agregar ni eliminar fluido, por lo tanto, todo el fluido que ingresa por un extremo de la tubería sale por el otro extremo. En otras palabras, la cantidad de fluido que entra es igual a la cantidad de fluido que sale.

Por otra parte, como el fluido es incompresible, su densidad es constante en todos sus puntos, de manera que el volumen de una porción del fluido que se traslada desde un lugar a otro dentro de la tubería, también se mantiene constante, aunque cambie su forma.

Analicemos esta situación considerando un pequeño intervalo de tiempo ( ∆t ). De acuerdo a la Figura 7.5, durante este intervalo, el fluido que entra a la tubería por el punto 1 recorre una distan- cia ∆x1 , mientras que el fluido que sale de la tubería por el punto 2, recorre una distancia ∆x2 . El volumen del fluido que entra es:

V A xentrada = ⋅1 1∆ (7.1)

Aunque puede parecer contradictorio, para un fluido ideal es una buena aproxima-ción usar el modelo de “partícula” para referirnos a pequeños elementos que lo componen. En rigor, nos referimos a porciones diminutas de masa del fluido, que imaginariamente se comportan como partículas individuales, de manera que el movimiento del fluido equivale al movimiento de un gran número de estos elementos.

Δx1

Δx2

A1

A2

v1

1

2

v2

Figura 7.5. Un fluido se desplaza de la parte baja del conducto hacia la parte alta, cambiando su sección trasversal. El volumen del fluido que pasa por la sección A1 en un tiempo Δt es igual al volumen del fluido que atraviesa la sección de área A2 en el mismo intervalo de tiempo.

Sección 7: Fluidos en movimiento

Page 53: FLUIDOS - FISICA

Física 3° Año Medio164

Fluidos

Por su parte, el volumen del fluido que sale de la tubería es:

V A xsalida = ⋅2 2∆ (7.2)

Sin embargo, como se trata de un fluido incompresible, el volumen que entra y el volumen que sale son iguales, es decir:

A x A x1 1 2 2⋅ = ⋅∆ ∆ (7.3)

Dividiendo ambos miembros de la ecuación (7.3) por el interva-lo de tiempo en el que se produce el desplazamiento del fluido, tenemos:

A x

tA x

t1 1 2 2⋅

=⋅∆

∆∆

∆ (7.4)

Cuando el intervalo de tiempo es muy pequeño, es decir, en el límite en que se aproxima a cero, el cuociente entre la distancia recorrida por el fluido y el intervalo de tiempo corresponde a la rapidez instantánea del fluido. Por lo tanto, la ecuación (7.4) se puede escribir como:

A v A v1 1 2 2⋅ = ⋅ (7.5)

Es decir, el producto del área por la rapidez del fluido es constante a lo largo de la tubería.

La ecuación (7.5) expresa cuantitativamente lo que habíamos comentado sobre las características de las líneas de flujo. Es de-cir, dado que la relación entre el área de la sección transversal de la tubería y la rapidez del fluido es inversamente proporcional, mientras mayor es el área, menor es la rapidez. En términos de líneas de flujo: cuando el área es mayor, más separadas se ven las líneas de flujo y viceversa.

La ecuación (7.5) se conoce como ecuación de continuidad de un fluido y es una expresión particular de la conservación de la masa. El producto del área por la rapidez del fluido, en cualquier punto del fluido a lo largo de la tubería, tiene unidades de volumen por

tiempo ms

3

y recibe el nombre de flujo de volumen o gasto.

Q = A · v (7.6)

¿Cómo se relaciona la ecuación de continuidad con el hecho de que el agua llegue más lejos al poner el pulgar sobre la punta de la manguera del jardín?

Figura 7.6. Si despreciamos la pérdida de agua por evaporación y por absorción de la tierra, en un canal de regadío se cumple la ley de conservación de la masa. Es decir, la masa de agua que entra al canal es igual a la masa de agua que sale.

Figura 7.7. Al inyectar una vacuna, el fluido contenido en la jeringa que es empujado lentamente por el émbolo, sale con mucha rapidez por el extremo de la aguja. ¿Por qué?

El flujo de volumen o gasto corresponde a la rapidez con la cual un volumen del fluido atraviesa una sección transver-sal. Por lo tanto, operacionalmente se expresa como:

QV

t=

∆ (7.7)

La ecuación 7.7 es equivalente a la ecuación 7.6.

Page 54: FLUIDOS - FISICA

165

Capítulo 4: Hidrodinámica

Ejemplo 1

El agua al interior de una manguera se comporta aproximada-mente como un fluido ideal. Consideremos una manguera de 2 cm de diámetro interno, por la que fluye agua a 0,5 m/s.

a) ¿Cuál es el flujo de volumen o gasto de agua que sale de la manguera?

b) ¿Cuál es la rapidez de salida del agua si en el extremo de la manguera se acopla otra manguera de 0,5 cm de diámetro?

a: De acuerdo a la definición de flujo de volumen, tenemos:

A v r v

A v mms

A v

⋅ = ⋅ ⋅

⋅ = ⋅( ) ⋅

⋅ = ⋅

π

π

2

2 2

4

10 0 5

1 57 10

,

,mms

3

Donde hemos considerado que el área de la sección transversal de la manguera tiene forma circular.

b: Al acoplar otra manguera de menor diámetro, esperamos que la reducción del área de la sección transversal au-mente la rapidez del flujo. En este caso, consideramos la ecuación (7.5):

A v A v

r v r v

mm

1 1 2 2

12

1 22

2

2 210 0 5

⋅ = ⋅

⋅ ⋅ = ⋅ ⋅

⋅( ) ⋅−

π π

π ,ss

m v

ms

v

ms

v

= ⋅ ⋅( ) ⋅

=

=

−π 0 25 10

0 50 25

8

2 2

2

2 2

2

,

,,

Es decir, como habíamos previsto, la rapidez de salida del agua aumenta 16 veces respecto a su rapidez de entrada.

Al abrir un poco una típica llave alta de lavaplatos, el flujo de agua sale a baja velocidad y su comportamiento es como el de un fluido estacionario. En estas condiciones se puede observar que el chorro de agua se estrecha a medida que desciende. ¿Por qué ocurre esto?

Figura 7.8. Con una boquilla en el extremo de la manguera se disminuye el área de la sección transversal por donde circula el agua y se consigue una mayor velocidad de salida del fluido.

Sección 7: Fluidos en movimiento

Page 55: FLUIDOS - FISICA

Física 3° Año Medio166

Fluidos

Ejemplo 2

a) En la Figura 7.9 se muestra la caída de un chorro de agua. Suponiendo que se trata de un flujo estacionario, ¿cuál es la rapidez de salida del agua?

a: Vamos a encontrar una expresión para determinar la velo-cidad de salida del agua en la boquilla de la llave, haciendo uso de la altura de caída y los diámetros inicial y final del flujo.

Cuando el flujo de agua sale de la boquilla con velocidad inicial vi, experimenta la acción de la fuerza de gravedad y podemos modelar el movimiento del agua como si se tratara de un flujo de “partículas” en caída libre. De acuerdo a esto, la velocidad final (vf) de las “partículas” de agua, cuando han caído una distancia (h), se puede obtener a partir de la siguiente expresión cinemática:

v v g hf i2 2 2= − ⋅ ⋅

Por otra parte, según la ecuación de continuidad para un fluido, tenemos:

A v A v

A vA

v

i i f f

i i

ff

⋅ = ⋅

⋅=

Reemplazando este resultado en la ecuación anterior, encon-tramos finalmente:

A vA

v gh

AA

v v

i i

fi

i

fii

= −

⋅ = −

2

2

2

2 2

2

2ggh

gh vAA

v

gh vA A

A

ii

f

if i

f

i2

2

2

2

2

22 2

2

= −

=−

=2

2

2 2gh

A

A Avf

f ii

Figura 7.9. En la medida en que el agua cae desde la llave disminuye el área de su sección transversal. ¿Por qué ocurre esto?

El resultado del Ejemplo 2 muestra que se puede obtener la rapidez de salida del agua a partir de parámetros observables.

Page 56: FLUIDOS - FISICA

167

Capítulo 4: Hidrodinámica

La ecuación de BernoulliEn general, la presión de un fluido cambia cuando se mueve por una zona en que cambia su rapidez o su altura sobre la superficie terrestre. Daniel Bernoulli, en el siglo XVIII, realizando experi-mentos con fluidos en movimiento, obtuvo por primera vez una ecuación que relaciona la presión con la rapidez y la elevación de un fluido ideal.

La ecuación de Bernoulli se puede derivar de las leyes de Newton. De hecho, es una forma distinta de expresar el teorema de con-servación de la energía mecánica. Veremos a continuación cómo es posible mostrar esto.

Consideremos la Figura 7.10, donde se muestra una porción de fluido que sube a través de una tubería. Para evaluar el cambio de energía mecánica experimentado por el fluido en su movimiento, fijamos nuestra atención en el sistema formado por la Tierra y el fluido contenido en la tubería entre los puntos 1 y 2.

Figura 7.10. Un fluido ideal sube a través de una tubería. El volumen del fluido que pasa por la sección A1 en un tiempo Δt es igual al volumen del fluido que atraviesa la sección de área A2 en el mismo intervalo de tiempo. El cambio de energía que tiene lugar en el sistema es equivalente a que si se moviera el fluido contenido en la sección de longitud Δx1 hasta la posición de la sección de longitud Δx2.

Figura 7.11. El río Baker, ubicado en la región de Aysén, es el río más caudaloso de Chile. Al bajar desde el el lago Bertrand, al sureste del lago General Carrera, su energía potencial gravitatoria se transforma en energía cinética. ¿Podrías aplicar la ecuación de Bernoulli al flujo del Baker?

Δx1

Δx2

A2

v1

1

v2

2

h1

h1F1 = P1 · A1

F2 = P2 · A2

Sección 7: Fluidos en movimiento

Page 57: FLUIDOS - FISICA

Física 3° Año Medio168

Fluidos

Aunque es evidente que el fluido se mueve a lo largo de toda la tubería, podemos observar que el cambio de energía que tiene lugar en el sistema, en un tiempo Δt, es equivalente a que si se moviera el fluido contenido en la sección de longitud Δx1 hasta la posición de la sección de longitud Δx2 (aunque en realidad sabemos que se trata de porciones diferentes del fluido).

Es decir, los elementos sombreados en la Figura 7.10 representan el único cambio que ocurre en el sistema entre la situación inicial y la situación final, ya que el resto del fluido comprendido entre estos dos elementos no experimenta ninguna variación.

Recordemos que, como vimos en la sección 3, el trabajo mecánico realizado sobre un sistema cambia la energía cinética o la energía potencial del sistema. Por esto, podemos escribir la ecuación de conservación de la energía del siguiente modo:

ΔEC + ΔEP = W (7.8)

Sobre el sistema que hemos definido de acuerdo a la Figura 7.10, actúan fuerzas externas en ambos extremos del fluido, que son aplicadas por el resto del fluido contenido en la tubería. Estas son las únicas fuerzas que actúan sobre el sistema y, por lo tanto, realizan trabajo sobre él, ya que se produce el desplazamiento de una masa de fluido.

Notemos que la fuerza de gravedad también actúa sobre el fluido, pero como hemos definido el sistema compuesto por la Tierra y el fluido, la fuerza de gravedad es una fuerza interna y, por con-siguiente, no realiza trabajo sobre el sistema.

A continuación, entonces, evaluaremos cada uno de los términos de la ecuación (7.8).

En primer lugar, consideremos el trabajo mecánico debido a las fuerzas externas. Por una parte, la fuerza F1

es aplicada en el

mismo sentido que el desplazamiento ∆x1

. Por otra parte, la

fuerza F2

es aplicada en sentido opuesto al desplazamiento ∆x2

.

Por lo tanto, en términos de módulos, el trabajo neto aplicado por ambas fuerzas es:

W F x F xW P A x P A x

= ⋅ − ⋅= ⋅ ⋅ − ⋅ ⋅

1 1 2 2

1 1 1 2 2 2

∆ ∆∆ ∆

(7.9)

Figura 7.12. Daniel Bernoulli (1700 -1782) nació en Groningen, Holanda. Fue uno de los notables matemáticos de la familia Bernoulli. Pero Daniel Bernoulli es particularmente recordado por sus aplicaciones de la matemática a la mecánica, especialmente a la mecánica de los fluidos. También fue pionero en su trabajo sobre probabilidad y estadística. El trabajo de Bernoulli es estudiado hoy en prácticamente todas las escuelas de ciencia del mundo.

Page 58: FLUIDOS - FISICA

169

Capítulo 4: Hidrodinámica

En la ecuación (7.9) podemos expresar el producto de la sección transversal por el desplazamiento como el volumen de los elemen-tos, de fluido correspondientes:

W P A x P A xW P V P VW P V

= ⋅ ⋅ − ⋅ ⋅= ⋅ − ⋅= ⋅

1 1 1 2 2 2

1 1 2 2

1

∆ ∆∆ ∆∆ −− ⋅P V2 ∆

(7.10)

Ya que el volumen de los elementos de fluido es el mismo (ΔV).

En segundo lugar consideremos el cambio de energía potencial gravitatoria experimentado por el sistema. La variación de la energía potencial gravitatoria en este caso está asociada al cambio de posición del elemento de fluido desde la posición 1 hasta la posición 2. Por lo tanto, tenemos:

∆E m g h m g hP = ⋅ ⋅ − ⋅ ⋅2 1 (7.11)

Donde m es la masa del elemento de fluido.

Por último, consideremos el cambio de energía cinética del sistema. Por supuesto, la única variación experimentada está relacionada con el movimiento del elemento del fluido en la posición 1 y en la posición 2, debido al cambio de rapidez. Como la masa del elemento de fluido es la misma en ambas posiciones, encontramos:

∆E m v m vC = ⋅ − ⋅12

122

212 (7.12)

Ahora podemos reescribir la ecuación (7.8), usando las relaciones (7.10), (7.11) y (7.12):

∆ ∆

E E W

m v m v m g h m g h P

C P+ =

⋅ − ⋅ + ⋅ ⋅ − ⋅ ⋅ = ⋅12

122

212

2 1 1VV P V− ⋅

2∆

Pero, dividiendo cada término por el volumen del elemento de fluido ΔV:

12

122

212

2 1 1⋅ ⋅ − ⋅ ⋅ + ⋅ ⋅ − ⋅ ⋅ = −

mV

vmV

vmV

g hmV

g h P∆ ∆ ∆ ∆

PP2

(7.14)

Pero notemos que el cuociente entre la masa y el volumen del elemento de fluido corresponde a su densidad.

(7.13)

Sección 7: Fluidos en movimiento

Page 59: FLUIDOS - FISICA

Física 3° Año Medio170

Fluidos

Por lo tanto, considerando la densidad del fluido en la ecuación (7.14), podemos escribir:

12

122

212

2 1 1 2⋅ ⋅ − ⋅ ⋅ + ⋅ ⋅ − ⋅ ⋅ = −ρ ρ ρ ρv v g h g h P P

Y reordenando los términos, tenemos:

P v g h P v g h1 1

21 2 2

22

12

12

+ ⋅ ⋅ + ⋅ ⋅ = + ⋅ ⋅ + ⋅ ⋅ρ ρ ρ ρ (7.15)

Esta es la ecuación de Bernoulli aplicada a un fluido ideal. En términos generales podemos escribir:

P v g h+ ⋅ ⋅ + ⋅ ⋅ =12

2ρ ρ constante (7.16)

De este resultado se puede inferir que la presión tiene que ver directamente con la energía. En rigor, la ecuación de Bernoulli es una expresión de la densidad de energía del sistema.

La ecuación de Bernoulli establece que la suma de la presión, la energía cinética por unidad de volumen y la energía potencial por unidad de volumen es constante en todos los puntos a los largo de una línea de flujo. Una consecuencia importante de ella es que los fluidos en movimiento rápido ejercen menos presión que los fluidos que se desplazan con lentitud (ver ejemplo 5).

Ejemplo 3

a) A partir de la ecuación de Bernoulli, demuestra que la pre-sión hidrostática se puede obtener como un caso particular de la presión de fluido en movimiento.

a: Consideremos la Figura 7.13. Aplicando la ecuación de Bernoulli a los puntos A y B, tenemos:

P v g h P v g hA A B B+ ⋅ + ⋅ ⋅ = + ⋅ + ⋅ ⋅

12

12

21

22ρ ρ ρ ρ

Sin embargo, en este caso el fluido está en reposo, por lo tanto:

P g h P g hP P g h g h

P P

A B

A B

A B

+ ⋅ ⋅ = + ⋅ ⋅= + ⋅ ⋅ − ⋅ ⋅

=

ρ ρρ ρ

1 2

2 1

++ ⋅ ⋅ −( )= + ⋅ ⋅

ρ

ρ

g h h

P P g hA B

2 1

Donde PB = P0, es decir, corresponde a la presión atmosférica.

Figura 7.13. La presión hidrostática de un fluido se puede obtener como un caso particular de la ecuación de Bernoulli.

A

B

h

h1

h2

atmósfera

Page 60: FLUIDOS - FISICA

171

Capítulo 4: Hidrodinámica

Ejemplo 4

Una piscina cilíndrica de plástico tiene un pequeño agujero en la parte inferior por donde escapa el agua. Se llena la piscina con agua potable hasta tener 1 m de profundidad, mientras que el agujero está a 20 cm del suelo.

a) ¿Cuál es la rapidez de salida del agua por el agujero?

b) ¿Cómo se puede expresar el flujo de volumen o gasto de agua que sale por el agujero en términos de su sección transversal (A) y las variables del problema?

c) ¿Cuál sería la rapidez de salida por el agujero si la piscina

se llenara con agua salada de densidad ρ = 1 0303

kgm

?

a: Podemos aplicar la ecuación de Bernoulli considerando que la presión en la superficie del agua y la presión en la salida por el agujero es igual a la presión atmosférica (P0). Además, como el agujero es relativamente pequeño en comparación con la piscina, podemos suponer que el nivel de agua desciende muy lentamente, con una rapidez despreciable (vsuperficie = 0). Así:

P v g h Po o+ ⋅ + ⋅ ⋅ = + ⋅12

12

2ρ ρ ρsuperficie superficie vv g h

v g h

agujero2

agujero

agujero2

s

+ ⋅ ⋅

⋅ = ⋅ ⋅

ρ

ρ ρ12 uuperficie agujero

agujero2

super

− ⋅ ⋅

= ⋅ ⋅

ρ g h

v g h2 fficie agujero

agujero superficie

−( )= ⋅ ⋅ −

h

v g h h2 aagujero( )Reemplazando la información dada en el enunciado:

vms

m m

vms

2 2

2

2 10 1 0 2

4

= ⋅ ⋅ −

=

( , )

b: El flujo de volumen o gasto de agua lo obtenemos como el producto del área de la sección transversal del agujero por la rapidez del flujo. Es decir, en término de las alturas involucradas:

A v A g h h⋅ = ⋅ ⋅ ⋅ −( )agujero superficie agujero2

Figura 7.14. Si los 3 agujeros en el recipiente tienen el mismo diámetro, ¿por qué varía el alcance de cada chorro de agua?

Sección 7: Fluidos en movimiento

Page 61: FLUIDOS - FISICA

Física 3° Año Medio172

Fluidos

4c: Si la densidad del fluido aumenta, la rapidez de salida no se modifica, ya que esta magnitud solo depende de la profundidad y no de la densidad del fluido.

Ejemplo 5

Una tubería horizontal de 10 cm de radio se reduce uniforme-mente hasta alcanzar una tubería de 5 cm de radio. Por su interior circula agua, cuya presión en la parte más ancha es de 8 x 104 Pa y en la parte más estrecha es de 6 x 104 Pa.

a) ¿Cuál es la rapidez del flujo de agua que circula por la tubería en la parte ancha y en la parte estrecha?

RB

hRAh

A B

Figura 7.15. Tubería horizontal con estrechamiento uniforme.

a: Consideremos los extremos A y B de la tubería en la Figura 7.15. Al aplicar la ecuación de Bernoulli en el centro del flujo en cada uno de esos puntos, tenemos:

P v g h P v g h

P v

A A B B

A A

+ ⋅ + ⋅ ⋅ = + ⋅ + ⋅ ⋅

+ ⋅

12

12

12

2 2

2

ρ ρ ρ ρ

ρ == + ⋅P vB B

12

Donde los términos relacionados con la elevación del fluido se cancelan, ya que la tubería está en posición horizontal, de manera que la altura (h) es la misma en ambos extremos.

Por otra parte, de la ecuación de continuidad obtenemos:

π π⋅ ⋅ = ⋅ ⋅

= ⋅

R v R v

vRR

v

A A B B

AB

AB

2 2

2

2

A continuación, reemplazamos este resultado en la ecuación de Bernoulli para calcular la rapidez del flujo en el punto B, es decir, en la sección más estrecha de la tubería.

Page 62: FLUIDOS - FISICA

173

Capítulo 4: Hidrodinámica

Reemplazando las datos del problema, encontramos:

P PRR

v

Pa Pa

A B

B

A

B

=

⋅ − ⋅

12

1

8 10 6 10

12

1

4

4

4 4

ρ

00 10 5 10

10

2 1

33

2 4

2 4

kgm

m

m

vB

−⋅( )

( )

=

,

0012

10 1 12

400 93

6 55

4

33 4

Pakgm

v

ms

v

B

B

=

=,

,mms

vB=

Y, por último, la rapidez del flujo en el punto A es:

vRR

v

vm

m

ms

v

AB

AB

A

= ⋅

=⋅( )

( )⋅

2

2

2 2

2 2

0 5 10

106 55

,,

AA

ms

= 1 63,

P v P v

PRR

v

A A B B

AB

AB

+ ⋅ = + ⋅

+ ⋅ ⋅

12

12

12

2 2

2

2

ρ ρ

ρ22

2

4

42 2

12

12

12

= + ⋅

+ ⋅ ⋅ = + ⋅

P v

PRR

v P v

P

B B

AB

AB B B

A

ρ

ρ ρ

−− = −

PRR

v

P PRR

BB

AB

A B

B

A

12

1

12

1

4

42

4

4

ρ

ρ

= vB

Sección 7: Fluidos en movimiento

Page 63: FLUIDOS - FISICA

Física 3° Año Medio174

Fluidos

Actividad de profundización ¿Cómo vuelan los pájaros, los coléopteros, las mariposas, los murciélagos y los aeroplanos?

Los pájaros, los coléopteros, las mariposas, los murciélagos y los aeroplanos, ne-cesitan alas para volar. Cuando los hermanos Wright lograron con éxito el primer vuelo a motor, ya habían pasado muchos años en el estudio de las alas. Responde en tu cuaderno las siguientes preguntas:

a) ¿Qué tienen en común las alas de los seres voladores? ¿En qué se asemejan a las alas de un avión?

Para realizar la siguiente actividad, se necesita lo siguiente: un rectángulo de car-tulina de 10 cm x 15 cm, tijeras, pegamento, una pajita de bebida, 30 cm de hilo y un secador de pelo.

Según la disponibilidad de secadores de pelo en el curso, reúnete con algunos com-pañeros y compañeras (4, idealmente) y formen un equipo de trabajo.

b) Reflexionen sobre esta pregunta: ¿qué tienen en común los seres voladores y los aviones que les permite volar? Como equipo, planteen una hipótesis para responder.

A continuación, realicen el siguiente experimento. Primero, doblen la cartulina en dos partes, dejando una solapa (lengüeta) de 2 cm. Unan la solapa con el borde del frente y péguenla, de modo que un lado del papel se curve. Con un lápiz, hagan un agujero en el ala en la parte de en medio y atraviesen la pajita por él. Recorten la pajita dejando 2 cm a cada lado del ala. Por último, atraviesen la pajita con el hilo, manténganlo en posición vertical y centren su modelo de ala, como se muestra en la imagen 7.4.

A continuación, dirijan el aire del secador hacia el borde doblado manteniéndolo a 3 cm del borde del ala. Varíen la inclinación del hilo de manera que la base del ala quede horizontal. Lentamente, muevan el secador hacia arriba y observen que sucede.

c) ¿Cómo se comporta el ala cuando recibe el aire del secador desde el frente? ¿Qué ocurre si el aire se dirige desde el lado opuesto?

d) Si el aire incide desde el frente, pero por debajo del ala, ¿se mantiene elevada?

e) ¿Cómo se relaciona el comportamiento del ala con el principio de Bernoulli?

Para finalizar la actividad, preparen un informe sobre su trabajo según las indica-ciones de su profesor(a) y luego presenten a sus compañeros(as) cuáles fueron sus hallazgos.

Imagen 7.4

Page 64: FLUIDOS - FISICA

175

Capítulo 4: Hidrodinámica

Evaluación intermediaPARTE I: Problema de planteamiento

1 Observa la siguiente imagen de la lateral:

En la figura se muestra un fluido que circula en régimen estacionario por un tubo de diámetro variable. v1 y v2 son la rapidez del fluido en los puntos 1 y 2. Análogamente, se indica la presión (P1 y P2), y el área transversal (A1 y A2) del conducto en ambos puntos. Por su parte, ρ es la densidad del fluido.

a) ¿Cómo se relacionan v1 y v2 con A1 y A2?b) ¿Cómo se relacionan P1 y P2 con v1 y v2?c) ¿Cómo relaciona la altura h con P1 y P2?

PARTE II: Análisis

2 Es bien sabido que el colesterol reduce la sección transversal del flujo de sangre en arterias y venas. Entonces, a partir del problema anterior, responde: ¿por qué es peligroso el exceso de colesterol?

Indagación N°20

¿Cómo cambia la rapidez un paracaidista desde que se lanza del avión hasta que abre el paracaídas?

Para responder la pregunta planteada en el título de esta actividad, se propone la siguiente hipótesis:

El paracaidista primero acelera de manera uniforme, luego desacelera uni-formemente y finalmente baja con velocidad constante.

¿Cómo podemos poner a prueba esta hipótesis?a) Junto a un compañero o una compañera, diseñen un procedimiento

experimental que les permita, a través de un modelo, poner a prueba la hipótesis para evaluar si es una explicación aceptable o debe ser descartada. Dibujen su montaje experimental y describan brevemente, pero con precisión, el procedimiento que sugieren.

Procuren que el procedimiento experimental propuesto sea factible de reali-zar en una hora de clases; es decir, que incluya el uso de materiales de fácil adquisición o construcción y tiempos razonables para la observación y el análisis de sus resultados.

b) Para finalizar, elaboren un informe de dos páginas según las indicaciones que les dé su profesor(a).

h

A1

v1 v2

P1 P2

A2 ρ

Sección 7: Fluidos en movimiento

Recuerda que un modelo es una representación simplificada del fenómeno que se intenta explicar, que incorpora sus principales características y, en especial, las variables medibles.

Page 65: FLUIDOS - FISICA

Física 3° Año Medio176

Fluidos

Aplicaciones de la ecuación de Bernoulli

EL MEDIDOR DE VENTURIEl efecto Venturi consiste en la disminución de la presión de un fluido cuando fluye a través de la sección reducida de una tubería. Como hemos visto, la rapidez del fluido aumenta en el tramo angosto de la tubería para satisfacer la ecuación de continuidad, mientras que su presión disminuye por la conservación de la energía.

El tubo de Venturi es una aplicación de este efecto presentada en el año 1797 por el físico italiano Giovanni Battista Venturi (1746-1822). Se trata de un dispositivo que sirve para medir la rapidez del flujo que lo atraviesa.

El medidor de Venturi consiste en un tubo de diámetro variable por el que circula el fluido. La diferencia de presión entre la región ancha y la región más estrecha puede medirse con un tubo vertical en forma de U conectando ambas regiones, el cual funciona como manómetro. Si se usa un líquido manométrico, como en el caso de la Figura 7.16, la diferencia de altura del líquido en las ramas del tubo en forma de U permite medir la diferencia de presión entre las dos secciones a las que está conectado.

flujo

manómetro

h

líquido manométrico

Figura 7.16. Tubo de Venturi. Un flujo de aire circula por el tubo de dia-metro variable. La diferencia de presión entre la región ancha y la región estrecha se puede obtener directamente midiendo la diferencia de altura entre las columnas del líquido manométrico.

Page 66: FLUIDOS - FISICA

177

Capítulo 4: Hidrodinámica

LA FUERZA DE SUSTENTACIÓNEl “efecto” de una pelota en el aire

Como muestra la Figura 7.17, una pelota que es lanzada en el aire como un proyectil rotando experimenta una fuerza de ascensión que es resultado de la diferencia entre la presión ejercida por el aire en la parte superior y en la parte inferior de la pelota.

Figura 7.18. La sustentación aerodiná-mica está comúnmente asociada con el ala fija de un avión. Sin embargo, la fuerza se sustentación también es generada por motores de propulsión; por rotores de helicóptero; por “alerones” de autos de carrera; por timones, velas y quillas de veleros; etc. Aunque el uso común de la palabra “sustentación” sugiere que se opone a la fuerza de gravedad, como se puede comprobar analizando el “efecto” de una pelota en el aire, la sustentación puede actuar en cualquier dirección.

Figura 7.17. Una pelota que se traslada a través del aire, de derecha a izquierda, rotando en el sentido de las agujas del reloj. Debido a la diferente velocidad relativa entre el aire y la superficie, que ocurre entre los dos puntos opuestos del objeto, se produce una diferencia de presión que tiene como resultado una fuerza neta dirigida hacia arriba. La presión es menor arriba de la pelota, donde la velocidad neta de la superficie respecto al aire es mayor.

¿Hacia dónde estaría dirigida la fuerza de sustentación si la pelota se lanzara rotando en sentido antihorario? ¿Y si la pelota rotara de manera que su eje de giro fuera perpendicular a la superficie terrestre, hacia dónde sería empujada por la fuerza de sustentación?

El vuelo de los aviones

Tal vez la aplicación más fascinante de la ecuación de Bernoulli sea el principio de sustentación del ala de un avión.

El diseño del perfil del ala de un avión genera trayectorias de longitud diferente para el flujo del aire que atraviesa, permitiendo una diferencia significativa en la forma que adquieren las líneas de flujo entre la parte superior del perfil alar y la parte inferior.

Aplicando la ecuación de Bernoulli a esta situación, se deduce que por la parte superior del ala el flujo tiene mayor rapidez que por la parte inferior; por lo tanto, la presión del aire es menor arriba que abajo, lo que genera una fuerza resultante que apunta en dirección ascendente.

Velocidad de traslación

Velocidad del aire a la pelota

Velocidad tangencial de la superficie

Velocidad neta de la superficie

Fuerza de sustentación

trayectoria

Sección 7: Fluidos en movimiento

Page 67: FLUIDOS - FISICA

Física 3° Año Medio178

Fluidos

Figura 7.19. El aire que se aproxima desde la izquierda es desviado por el ala del avión de manera que las líneas de flujo están más juntas en la parte superior del ala que en la parte inferior. Como la corriente de aire es desviada por el avión, esta corriente ejerce una fuerza F

sobre

el ala, cuya componente vertical es la fuerza de sustentación, debida a la diferencia de presión del aire sobre las superficies superior e inferior del ala. La componente horizontal de F

es la resistencia del aire al

movimiento del avión.

De acuerdo a la Figura 7.19, el aire ejerce una fuerza F

sobre el ala del avión. Esta fuerza tiene una componente vertical que corres-ponde a la fuerza de sustentación o sustentación aerodinámica, y una componente horizontal llamada resistencia.

En general, la fuerza de sustentación depende de varios factores, como la rapidez del avión, el área del ala y su curvatura, así como del ángulo que forma el ala con la horizontal, llamado ángulo de ataque. Cuando este ángulo aumenta, el flujo por encima del ala puede volverse turbulento, disminuyendo la sustentación.

Viscosidad y velocidad límiteAl inicio de esta sección indicamos que la viscosidad es una propiedad de los fluidos relacionada con la fricción interna que experimentan sus capas, de manera que cuando un fluido es más viscoso hay mayor fricción entre sus capas, lo que dificulta su movimiento, de manera análoga a la acción de la fuerza de roce por deslizamiento entre dos superficies. Sin embargo, en nuestro modelo de fluido ideal la viscosidad no está presente.

sustentación

resistencia

F

Page 68: FLUIDOS - FISICA

179

Capítulo 4: Hidrodinámica

En realidad, todos los fluidos conocidos presentan algo de viscosi-dad, aunque como hemos visto el modelo de viscosidad nula es una aproximación bastante buena para ciertas aplicaciones. A continuación, consideraremos algunas características de un fluido viscoso.

La viscosidad se puede explicar por la fuerza de cohesión entre las moléculas del fluido. Por lo tanto, depende del estado físico y de la temperatura del fluido.

Así, los gases, por ejemplo, tienen menor viscosidad que los lí-quidos. La miel es más viscosa que el aceite, ya que su cohesión es mayor; y por la misma razón, la leche es más viscosa que el agua. Sin embargo, al calentar miel se puede observar como su viscosidad disminuye notablemente.

Para comprender cómo la viscosidad afecta el movimiento de un fluido en régimen estacionario o laminar, sin turbulencia, supone-mos que está constituido por capas. De acuerdo a esto, cuando un fluido viscoso se mueve a través de una tubería sus capas externas interactúan con las paredes del conducto, lo que provoca un mo-vimiento diferente a las capas internas, ya que la viscosidad actúa retrasando a las capas externas.

En estas condiciones, el fluido experimenta una baja de presión a lo largo de su dirección de movimiento, que depende directamente del coeficiente de viscosidad del fluido.

Figura 7.21. Típicamente, al vertirse leche sobre agua se producen salpica-duras. Fluidos con viscosidad elevada no producen tales efectos al caer con una rapidez similar.

Figura 7.22. Un fluido viscoso en ré-gimen estacionario se arrastra lámina por lámina. La viscosidad del fluido provoca que sus capas externas, que interactúan con las paredes de la tube-ría, se retrasen respecto a las capas internas, de manera que la rapidez del fluido es mayor en el centro.

Figura 7.20. Un fluido ideal no presenta viscosidad, por lo que su presión se mantiene constante a lo largo de su movimiento al interior de la tubería. En cambio, un fluido viscoso pierde presión en la medida que avanza.

La ley de Poiseuille permite determinar el flujo de volumen (Q) de un fluido en régimen estacionario o laminar, incompresible y uniformemente viscoso, a través de un tubo cilíndrico de sección circular constante, como el de la Figura 7.20. Esta ley se expresa matemáticamente del siguiente modo:

QP P r

L=

−( ) ⋅ ⋅⋅ ⋅

1 24

η (7.17)

P Fluido ideal Fluido viscosoP1P P2

pared de la tubería

pared de la tubería

vinferior

vexterior

Sección 7: Fluidos en movimiento

η es la letra griega “eta”.

Page 69: FLUIDOS - FISICA

Física 3° Año Medio180

Fluidos

En la ecuación (7.17), Q es el flujo de volumen o gasto de fluido que circula por la tubería, P1 y P2 corresponden a la presión del fluido en dos puntos diferentes que están separados por una dis-tancia L; r es el radio interno del cilindro y η es el coeficiente de viscosidad que en el S.I. se expresa en [Pa · s]

De acuerdo a la ecuación de Bernoulli, la presión es una medida de la densidad de energía de un fluido. ¿Cómo se relaciona la viscosidad con la energía que transporta un fluido? ¿Podemos aplicar la ecuación de Bernoulli a un fluido viscoso?

Cuando un cuerpo se mueve en el seno de un fluido viscoso, la resistencia que presenta el medio depende de la forma del cuerpo y de la velocidad relativa entre el cuerpo y el fluido. El régimen de flujo estacionario o laminar se mantiene mientras la velocidad relativa es inferior a cierto valor crítico, y en este caso la resistencia que ofrece el medio es debida casi exclusivamente a las fuerzas de fricción que se oponen al resbalamiento de las capas de fluido.

Un caso particular es el de un objeto esférico que se mueve en el interior de un fluido viscoso. Este caso especial fue estudiado por Stokes, a mediados del siglo XIX, quien derivó una relación para la fuerza de resistencia experimentada por el objeto. Esta relación es conocida como ley de Stokes y se expresa como:

F r v= ⋅ ⋅ ⋅ ⋅6 π η (7.18)

Donde η es el coeficiente de viscosidad del fluido, r es el radio del objeto esférico y v es su rapidez. En general, la ley de Stokes es válida en el caso de partículas esféricas pequeñas, moviéndose a velocidades bajas.

Consideremos el caso de un objeto esférico que se deja caer al interior de un fluido, representado en la Figura 7.23. El objeto cae acelerado hasta que las tres fuerzas que actúan sobre él se equi-libran entre sí, de manera que el movimiento se vuelve uniforme y mantiene una velocidad límite. A continuación, derivaremos una expresión para el módulo de la velocidad límite, usando la ley de Stokes.

Cuando el objeto alcanza un movimiento uniforme, el equilibrio de fuerzas se expresa, en términos de módulos, como:

E + F – P = 0 (7.19)

Como el fluido viscoso presenta una distribución de velocidades entre sus distintas capas, en la ecuación (7.17) podemos expresar el flujo de volumen (Q) en términos de la rapidez media del fluido (vmedia) y del área de la sección transversal de la tubería (A):

Q A media= ⋅υ (7.20)

Figura 7.23. Un objeto que se deja

caer al interior de un fluido viscoso

experimenta la acción de tres fuerzas:

el empuje ( E

), la fuerza de resistencia

( F

) y el peso ( P

).

E

F

v

P

fluido

Page 70: FLUIDOS - FISICA

181

Capítulo 4: Hidrodinámica

Como se muestra en la Figura 7.23, en la ecuación (7.17) las fuer-zas involucradas son el empuje ( E

), la fuerza de resistencia ( F

)

y el peso del objeto ( P

). Por lo tanto, considerando la densidad del objeto (ρ0) y su volumen (V), podemos escribir:

E F P

V g r v V gf o

+ − =⋅ ⋅ + ⋅ ⋅ ⋅ ⋅ − ⋅ ⋅ =

06 0ρ π η ρ (7.21)

Donde ρf es la densidad del fluido. Al reemplazar el volumen por el de una esfera, obtenemos:

ρ π η ρ

ρ π π η

f o

f

V g r v V g

r g

⋅ ⋅ + ⋅ ⋅ ⋅ ⋅ − ⋅ ⋅ =

⋅ ⋅ ⋅ + ⋅ ⋅ ⋅

6 0

43

63 rr v r go⋅ − ⋅ ⋅ ⋅ =ρ π43

03 (7.22)

Es decir:

6

43

43

29

3 3

2

⋅ ⋅ ⋅ ⋅ = ⋅ ⋅ ⋅ − ⋅ ⋅ ⋅

= ⋅⋅

π η ρ π ρ π

η

r v r g r g

vr g

o f

⋅⋅ −( )ρ ρo f

(7.23)

Donde v es la velocidad límite que alcanza el objeto en su caída a través del fluido.

¿Cómo afectaría a las personas la caída de agua durante la lluvia si las gotas no experimentaran la resistencia del aire?

Ejemplo 6

Una esfera de aluminio de 2 mm de diámetro, cae en el interior de un tiesto con glicerina a 20°C. La densidad del aluminio es ρ0

33

2 7 10= ⋅,kgm

y la densidad de la glicerina es ρ f

kgm

= ⋅1 26 1033

, . Además, el coeficiente de viscosidad de la glicerina a esta temperatura es η = ⋅1 49, Pa s

a) ¿Qué velocidad alcanza la esfera?

b) ¿Cuál es el módulo de la fuerza de resistencia que actúa sobre la esfera cuando se mueve con velocidad constante?

Figura 7.24. La velocidad de caída de un cuerpo en el interior de un fluido viscoso aumenta hasta alcanzar la velocidad límite.

Rap

idez

(m/s

)

Tiempo (s)

Figura 7.25. Al llover, las gotas de agua caen al suelo con una velocidad límite de aproximadamente 7 m/s.

Sección 7: Fluidos en movimiento

Page 71: FLUIDOS - FISICA

Física 3° Año Medio182

Fluidos

a: De acuerdo a la ecuación (7.21), la velocidad límite es:

v

m ms

Pa skgm

=( ) ⋅

⋅⋅ ⋅ −

29

10 10

1 492 7 10 1 2

3 2

23

3,( , , 66 10

2 15 10

33

3

= ⋅ −

kgm

vms

)

,

b: Para encontrar la fuerza de resistencia, usamos el resultado anterior para evaluar ley de Stokes:

F r v

F Pa s m

= ⋅ ⋅ ⋅ ⋅

= ⋅ ⋅ ⋅( ) ⋅( ) ⋅−

6

6 3 14 1 49 10 2 153

π η

, , , ⋅⋅

= ⋅

10

6 04 10

3

5

ms

F N,

El flujo sanguíneo en el cuerpo humanoNuestro corazón es análogo a una máquina que funciona como una bomba para mover fluidos. En este caso, el fluido es la sangre. El ritmo de la frecuencia cardíaca del corazón marca los movimientos de contracción y dilatación periódicos de esta bomba natural. El proceso de contracción se denomina sístole, y el de expansión o relajación, diástole.

El flujo sanguíneo que sale del corazón comienza su camino por el cuerpo saliendo por la arteria aorta para distribuirse por el resto de las arterias que recorren las distintas zonas corporales, llegando a distribuirse a todas las células del cuerpo a través de la extensa red de vasos capilares. En el proceso de regreso, la sangre se mueve a través de las venas, para volver al corazón a través de la vena cava.

Durante la sístole la presión sanguínea es máxima. En cambio durante la diástole, la presión del flujo sanguíneo disminuye. Por esta razón, las presiones generadas durante un ciclo cardíaco se denominan presión sistólica y presión diastólica.

Existen varios procedimientos para medir la presión arterial. Uno de ellos hace uso de un instrumento conocido como esfig-momanómetro, con el cual se puede medir la presión arterial de manera indirecta, ya que se comprimen externamente la arteria y los tejidos adyacentes, y se supone que la presión necesaria para ocluir la arteria es igual a la que hay dentro de ella.

Figura 7.26. El sistema circulatorio humano consta de un generador de pulsos de presión o bomba –el corazón–, un sistema para captación de oxígeno y expulsión de desechos – los pulmones–, un medio portador de oxígeno y otros nutrientes –la san-gre–, y un sistema de distribución –la red de arterias, venas y capilares en todo el cuerpo. La tarea principal del sistema circulatorio es el transporte de oxígeno y dióxido de carbono desde y hacia el sistema de intercambio con el medio (los pulmones). El corazón es un órgano prodigioso en muchos sentidos, pero desde un punto de vista físico, es interesante porque resuelve el siguiente problema: si la bomba se usa para generar presión y hacer llegar la sangre al sistema de intercambio, queda poca presión para distribuir la sangre oxigenada a los tejidos. Y en cambio, si la bomba se usa para ge-nerar presión para hacer llegar sangre a los tejidos, queda poca presión para impulsar la sangre desoxigenada al sistema de intercambio. El corazón es la solución que la naturaleza encontró para esto: una bomba doble.

CerebroTiroides

Venatiroidea

Pulmones

Arteriatiroidea

Vena cava superior

Arteria aorta

Vena cava inferior

Hígado

Bazo

Intestino

Venas hepáticas

Vena porta

Arteria hepática

Arteria mesentérica

superior

Riñón

Vena renal Arteriarenal

Venapulmonar

Page 72: FLUIDOS - FISICA

183

Capítulo 4: Hidrodinámica

El esfigmomanómetro consiste en una cámara elástica con forma de brazalete, la cual se infla con aire hasta una presión suficiente para eliminar temporalmente los pulsos cardíacos en la zona estrangulada, lo que se puede constatar escuchando a través de un estetoscopio. La presión del aire se mide con un manómetro de mercurio.

Figura 7.28. La sístole y la diástole no ocurren simultánemente en todas las cavidades del corazón. (a) Sístole auri-cular. (b) Sístole ventricular. (c) Diástole auricular (d) Diástole ventricular.

Figura 7.27. Un esfigmomanómetro consiste en un manómetro junto a una cámara con aire y un estetoscopio.

Cuando se baja controladamente la presión del aire en el brazalete, la circulación sanguínea se activa nuevamente en la arteria oclui-da, escuchándose claramente los bombeos del corazón a través de las pulsaciones. La presión indicada por el manómetro en el instante en que se escucha la primera pulsación de reactivación es la presión sistólica.

Luego de esta medición, se continúa sacando aire de la cámara paulatinamente, de manera que la presión disminuye lo suficiente para que el flujo de sangre se restituya por completo. En el ins-tante en que esto ocurre, el sonido de las pulsaciones a través del estetoscopio deja de ser nítido, y entonces se vuelve a registrar el valor de la presión indicado por el manómetro. El valor medido en este caso corresponde a la presión diastólica.

Los valores de presión sistólica y presión diastólica normales son de 120 mmHg y 80 mmHg, respectivamente. Sin embargo, estos valores fluctúan entre 100 y 140 mmHg, para la presión sistólica y entre 70 y 90 mm Hg para la presión diastólica.

¿Qué efecto sobre el flujo sanguíneo produce el colesterol que se deposita en las arterias?

aortaaurículaarteria pulmonar

contracción ventrículo

(a)

aortaaurículaarteria pulmonar

ventrículo

contracción (b)aortaaurícula

arteria pulmonar

ventrículo

(c)aortaaurícula

arteria pulmonar

ventrículo

(d)

Sección 7: Fluidos en movimiento

Page 73: FLUIDOS - FISICA

Física 3° Año Medio184

Fluidos

A partir de la lista de conceptos relevantes (CR) y frases conectoras (FC), completa en tu cuaderno el mapa conceptual de la figura.

Conceptos Relevantes (CR) Frases Conectoras (FC)A Rapidez de flujo I Se deriva delB Ecuación de continuidad II Expresa que es constante elC Fluidos incompresibles III Cumplen laD Movimiento IV Estudio deE Fuerza de sustentación V Es el producto entreF Flujo estacionarioG Líquidos

Síntesis

DesafíoCuando hayas terminado esta

actividad, vuelve a leer el texto de la sección, con mucha aten-ción, y genera tu propio mapa

conceptual.8

Tubo de Venturi

1

Flujo sanguíneo

Explica fenómenos como

Gases 3 tienen Viscosidad

En

2

11

Ecuación de Bernoulli9

Principio de conservación de la

energía

6Flujo de volumen

y también la

4

Flujos irrotacionales

5

Fluidos no viscosos

Área de sección

7

12

Siempre que sean

Hidrodinámica

10

Page 74: FLUIDOS - FISICA

185

Capítulo 4: Hidrodinámica

Preguntas y ejercicios propuestos

1 ¿Qué aspecto de los trabajos de Daniel Bernoulli ha sido destacado en el transcurso de los siglos, al punto que todavía hoy no hay escuela de in-geniería, física, medicina o construcción, entre otras, donde no se estudien sus resultados?

2 Observa las situaciones representadas en la Figura 7.29. Realiza estos experimentos y luego explica lo que observas relacionándolo con tu conocimiento sobre la dinámica de los fluidos. (a) Soplar por encima de una hoja de papel dis-puesta horizontalmente bajo la boca. (b) Soplar entre dos hojas de papel dispuestas verticalmente. (c) Con un secador de pelo, mantener una pelota de ping-pong en equilibrio. (d) Sostener suavemente con un dedo una pelota de ping-pong dentro de un embudo y luego soplar por el extremo del-gado del embudo, mientras se saca el dedo que sostenía a la pelota.

Figura 7.29

3 Por una manguera de 2 cm de diámetro fluye gasolina, en régimen estacionario, con una ra-pidez de 5 m/s. (a) ¿Cuál es el flujo de volumen o gasto, expresado en m

s

3

? (b) ¿Cuánto tiempo se requiere para llenar con este flujo un tanque de 100 m3?

4 En la salida de una manguera de 3 cm de diá-metro, fluye agua con una rapidez de 2 cm/s. (a) ¿Cuál es el flujo de volumen expresado en l

min?

(b) ¿Cuánto tiempo se requiere para llenar con este flujo un recipiente de 50 litros?

5 Por una tubería horizontal de 20 cm de diámetro fluye agua con un flujo de volumen de 30

3mmin

. (a) ¿Con qué rapidez sale el agua en la boca de la tubería? (b) Si la tubería está situada a 1,2 m de altura sobre el suelo, como se muestra en la Figura 7.30, ¿qué alcance horizontal (x) tiene la corriente de agua desde la salida?

Figura 7.30

6 Un recipiente para guardar agua, abierto a la at-mósfera por su parte superior, tiene un pequeño orificio en la parte inferior, a 6 m por debajo de la superficie del líquido. (a) ¿Con qué rapidez sale agua por el orificio? (b) Si el área del orificio es de 1,3 cm2, ¿cuál es el gasto de agua que sale por del recipiente?

(a) (b)

(c) (d)

Sección 7: Fluidos en movimiento

1,2 m

x

Page 75: FLUIDOS - FISICA

Física 3° Año Medio186

Fluidos

7 El agua fluye con un gasto de 6 m3/min, a través de una pequeña abertura en el fondo de un gran tanque cilíndrico, que está abierto a la atmósfera en la parte superior. El agua del tanque tiene 10 m de profundidad. (a) ¿Con qué rapidez sale el chorro de agua por la abertura? (b) ¿Cuál sería el gasto de la fuga de agua, si se aplica una presión adicional equivalente a 3

4 del valor de la presión

atmosférica?

8 Un flujo de agua en régimen estacionario circula por una tubería horizontal. En un punto donde la presión absoluta es de 300 kPa, la rapidez es de 2 m/s. (a) ¿Con qué rapidez fluye el agua en un sector donde la tubería se estrecha, de modo que la presión del fluido se reduce a 100 kPa?

9 Un flujo agua en régimen estacionario circula por una tubería horizontal de sección transversal variable. En los puntos donde el área de la sección transversal es de 10-2 m2, la presión del fluido alcanza los 5·105 Pa y su rapidez es de 0,5 m/s. (a) ¿Cuál es la rapidez del fluido en una región constreñida de la tubería, donde el área de la sección transversal es de 4 · 10-4 m2? (b) ¿Cuál es la presión a la que pasa el agua por esta zona estrecha?

10 Un gran tanque de almacenamiento se llena con agua pura. (a) Sin tener en cuenta la viscosidad, demuestra que la rapidez del agua que sale por un agujero, en uno de los lados del tanque, a una distancia de h por debajo de la superficie del agua, es v g h= ⋅ ⋅2 . Este resultado fue derivado inicialmente por Torricelli, de ahí que se conoce como teorema de Torricelli.

11 A partir del resultados del ejercicio 10, (a) demuestra que se obtiene el máximo alcance del agua que sale por el agujero cuando la posición del orifi-cio coincide con la mitad de la profundidad del líquido, es decir, a la mitad de la distancia entre la superficie del líquido y el fondo del tanque.

12 Una manguera de jardín de 15 m de largo y diámetro interior de 1,25 cm, en posición hori-zontal, se utiliza para suministrar agua a 20°C (η = 1,003·10−3 Pa·s), la cual circula en la man-guera a una rapidez media de 150 cm/s. (a) ¿Cuál es la diferencia de presión entre los extremos de la manguera, debido a la viscosidad?

13 Fluye mercurio a 25°C (η = 1,526·10−3 Pa·s) por un tubo de 4 cm de diámetro y 0,5 m de largo. Si la diferencia de presión de un extremo a otro del tubo es igual a 104 Pa (alrededor de 1/10 atm), (a) ¿cuál es la rapidez media del fluido al interior del tubo?

14 Una bola de acero de 8 mm de diámetro se suelta dentro de un cilindro que contiene glicerina a 25°C. (η = 1,49 Pa·s) (a) ¿Cuál es la velocidad límite de la bola? ( ρacero = ⋅7 8 103

3,

kgm

,ρglicerina = ⋅1 26 1033

,kgm

)

15 En una botella recién sacada del refrigerador, que contiene un fluido desconocido, a una temperatura de 5° C, se deja caer una bolita, cuyo diámetro es de 1,57 cm y cuya densidad es de 2 5 103

3, ⋅

kgm

. La bolita cae en todo su movimiento con rapi-dez constante y tarda 45 s en hundirse hasta el fondo. La profundidad del líquido es de 12,1 cm y su densidad es de 1 2 103

3, ⋅

kgm

. (a) ¿Cuál es el coeficiente de viscosidad del fluido a 5° C? (b) Cuando se realiza el mismo experimento con el fluido a temperatura ambiente, la bollita cae en similares condiciones, pero demora 5 s en llegar al fondo. ¿Cuál es la viscosidad del fluido a tem-peratura ambiente?

Page 76: FLUIDOS - FISICA

187

Capítulo 4: Hidrodinámica

Evaluación final de la sección

PARTE I: Anota en el recuadro el número del concepto que corresponde a su descripción o definición.

Magnitud Unidad1 Densidad Cada partícula del fluido sigue una trayectoria denominada

línea de flujo, que no se cruza con las demás.2 Flujo laminar o estacionario Masa del fluido por unidad de volumen.3 Flujo turbulento Las trayectorias de las partículas del fluido pueden formar

remolinos o cruzarse entre sí.4 Viscosidad Fricción interna del fluido.

PARTE II: Indica si el enunciado es verdadero o falso. Expresa en tu cuaderno la justificación de tus respuestas.

V o F1 La rapidez de caída libre de un objeto en un fluido alcanza un límite determinado.2 La fuerza de sustentación empuja hacia adelante a un avión.3 Un fluido estacionario horizontal se mueve con mayor rapidez cuando mayor es su área trans-

versal.4 La presión sistólica es menor que la presión diastólica.5 El esfigmomanómetro permite medir la presión de la sangre en las arterias.

PARTE III: Responde las siguientes preguntas, marcando la alternativa correcta.

1 El colesterol se acumula en las arterias y venas, reduciendo la sección transversal de la sangre. ¿cómo afecta esto a la presión y rapidez de la sangre?a) La presión aumenta, la rapidez disminuye.b) La presión aumenta, la rapidez aumenta.c) La presión disminuye, la rapidez disminu-

ye.d) La presión disminuye, la rapidez aumenta.

2 Un futbolista le pega a una pelota con “chanfle”, utilizando el borde externo de su pie izquierdo. Justo después del golpe, la pelota sale hacia adelante y rotando con sentido antihorario (vista desde arriba), en torno a un eje vertical. ¿Hacia dónde diría el futbolista que apunta la fuerza de sustentación que actúa sobre la pelota? a) Hacia adelante. b) Hacia su derecha. c) Hacia su izquierda.d) Hacia arriba.

3 Una piscina de 10 m3 tiene un agujero de 10 cm de diámetro en el fondo, y el agua sale a una rapidez de 10 m/s. ¿En cuánto tiempo se vacía? a) 2,12 minutos. b) 0,53 minutos. c) 31,83 segundos. d) 2,12 segundos.

4 Una esfera de aluminio cae en el interior de un tiesto con glicerina. Si la esfera tiene 1 mm de radio y cae a una velocidad límite de 0,0215 m/s, ¿cuál es la rapidez que alcanza si su radio au-menta a 2 mm?a) 0,043 m/s b) 0,086 m/s c) 0,011 m/sd) 0,005 m/s

Sección 7: Fluidos en movimiento